Conceptual Framework Flashcards
Which of the following related party transactions by a company should be disclosed in the notes to the financial statements?
Payment of per diem expenses to members of the board of directors
Consulting fees paid to a marketing research firm, one of whose partners is also a director of the company
A. I only
B. II only
C. Both I and II
D. Neither I nor II
The correct answer is: B
Explanation:
Financial statements should include disclosures of material related party transactions, other than compensation arrangements, expense allowances, and other similar items in the ordinary course of business. Therefore, the consulting fees paid to a director must be disclosed in the notes to the financial statements. The per diem expenses would be included in expense allowances and would not need to be disclosed.
In general, an enterprise preparing interim financial statements should
A.
Defer recognition of seasonal revenue
B.
Disregard permanent decreases in the market value of its inventory
C.
Allocate revenues and expenses evenly over the quarters, regardless of when they actually occurred
D.
Use the same accounting principles followed in preparing its latest annual financial statements
The correct answer is: D
Explanation:
Each interim period should be viewed as an integral part of an annual period and not as a separate, independent period. In order to maintain comparability between interim and annual financial statements, the principles and practices used to prepare the latest annual financial statements are also to be used in preparing the interim statements. Revenue should be recognized as earned during an interim period on the same basis as followed for recognition of income for the full year. Costs and expenses associated directly with products sold or services rendered for annual reporting purposes generally should be similarly treated for interim reporting purposes.
Super Seniors is a not-for-profit organization that provides services to senior citizens. Super employs a full-time staff of 10 people at an annual cost of $150,000. In addition, two volunteers work as part-time secretaries replacing last years’ full-time secretary who earned $10,000. Services performed by other volunteers for special events had an estimated value of $15,000. These volunteers were employees of local businesses and they received small-value items for their participation. What amount should Super report for salary and wage expenses related to the above items?
A. $150,000 B. $160,000 C. $165,000 D. $175,000
The correct answer is: B
Explanation:
Other not-for-profit organizations (ONPOs) should report donated services as support and expense if the following conditions are met: (1) the services are a normal part of the program or supporting services and would otherwise be performed by salaried personnel, (2) the organization exercises control over the employment and duties of the donors of the services, (3) the ONPO has a clearly measurable basis for the amount, (4) the services are significant, and (5) the services of the ONPO are not primarily for the benefit of its members. Since all of the above conditions are met for the part-time secretaries, the $10,000 estimated value of the donated secretarial services plus the $150,000 annual cost of its full-time staff for a total of $160,000 should be reported for salary and wages expense. The $15,000 estimated value of volunteer work for special events does not meet the criteria to be recognized as an expense.
Tam Co. reported the following items in its year-end financial statements:
Capital expenditures $1,000,000 Finance lease payments 125,000 Income taxes paid 325,000 Dividends paid 200,000 Net interest payments 220,000 What amount should Tam report as supplemental disclosures in its statement of cash flows prepared using the indirect method?
A. $545,000 B. $745,000 C. $1,125,000 D. $1,870,000
The correct answer is: A
Explanation:
Net cash flow from operating activities may be reported under the indirect method by adjusting net income to reconcile it to net cash flow from operating activities.
The reconciliation of net income to net cash flow from operating activities should separately report all major classes of reconciling items. In addition, when the direct method is used, amounts of interest paid (net of amounts capitalized) and income taxes paid during the period should be provided in related disclosures.
The $200,000 dividends paid is not included in the supplemental disclosures. The $325,000 income taxes paid plus the $220,000 net interest payments total $545,000.
Dunbarn Co. had the following activities during the year:
Purchase of inventory $120,000
Purchase of equipment $80,000
Purchase of available-for-sale debt securities $60,000
Purchase of treasury stock $70,000
Issuance of common stock $150,000
What amount should Dunbarn report as cash provided (used) by investing activities in its statement of cash flows for the year?
A. $(120,000) B. $(140,000) C. $(210,000) D. $150,000
The correct answer is: B
Explanation:
The correct answer is Option (B).
Cash flows from investing activities represent the extent to which expenditures have been made for resources intended to generate future income and cash flows. Cash flows from investing activities include cash payments for property, plant, and equipment, other long-lived assets, equity and debt instruments held for investment purposes, and cash advances and loans made to other parties. Net cash flow used in investing activities will be reported at ($140,000).
Purchase of equipment ($80,000)
Purchase of available-for-sale debt securities ($60,000)
Net cash provided (used) by investing activities ($140,000)
(A) is incorrect because the purchase of inventory is cash flow used in operating activities.
(C) is incorrect because the purchase of treasury stock is cash flow used in financing activities and not part of investing activities.
(D) is incorrect because the issuance of common stock is a cash inflow from financing activities.
How should unconditional pledges received by a nongovernmental not-for-profit organization that will be collected over more than one year be reported?
A.
Long-term pledges receivable, valued at the expected collection amount
B.
Pledges receivable, valued at their present values
C.
Deferred revenue, valued at present value
D.
Pledges receivable, valued at the amount pledged
The correct answer is: B
Explanation:
Unconditional pledges are reported as a receivable at their present values in the period in which they are made, net of an allowance for uncollectible amounts. They are not recorded as long-term, deferred revenue, or at the full amount pledged.
Net Income Formula
Gross Income- Expenses= Net Income
OR
Revenue-COGS-Expenses= Net Income
Basic Earning Per share Formula
Net Income – Preference Dividend
_______________________
#number of issued shares
Diluted EPS Formula
Net Income- Prefered Dividend +Paid Out to Dilutive Securities
_________________________________________
Weighted average Outstan. Shares + Converted Dilutive Securities
A company is preparing its year-end cash flow statement using the indirect method. During the year, the following transactions occurred:
Dividends paid $300
Proceeds from the issuance of common stock 250
Borrowings under a line of credit 200
Proceeds from the issuance of convertible bonds 100
Proceeds from the sale of a building 150
What is the company’s increase in cash flows provided by financing activities for the year?
A. $50 B. $150 C. $250 D. $550
The correct answer is: C
Explanation:
Included in the financing activities are cash effects of (1) obtaining resources from owners and providing them with a return on their investment,(2) short and long term borrowings and repaying amounts borrowed,or otherwise settling the obligation, (3) obtaining and paying for other resources obtained from creditors on long-term credit, and (4) derivatives that contain a financing component and are accounted for a fair-value or cash-flow hedges.Proceeds from the sale of a building are an investing activity.
Dividends paid ($300)
Proceeds from the issuance of common stock 250
Borrowings under a line of credit 200
Proceeds from the issuance of convertible bonds 100
Net Cash provided from financing activities $250
Option (a) is incorrect because increase in cash flow from financing activities is reported at $250.Option (b) is incorrect because proceeds from issuance of convertible bonds are not considered ($150 = $250 + $200 -$300). Option (d) is incorrect because dividends paid should be reported as use of cash in financing activities ($550 = $250 + $200 + $100).
Each of the following events is required to be reported to the United States Securities and Exchange Commission on Form 8-K, except
A.
The creation of an obligation under an off-balance sheet arrangement of a registrant.
B.
The unregistered sale of equity securities.
C.
A change in a registrant’s certifying accountant.
D.
The quarterly results of operations and financial condition of a registrant.
The correct answer is: D
Explanation:
The quarterly results of operations and financial condition of a registrant would be reported to the SEC on the Form 10-Q, not the Form 8-K. The Form 8-K is used to report “current events” so that investors can obtain information about material events in a timely manner rather than waiting until the next quarterly or annual information is filed. Common events reported include quarterly earnings releases, notification about entering into material agreements (including mergers), and entering into debt or other direct financial obligations. This would include the creation of an obligation under an off-balance sheet arrangement of a registrant, the unregistered sale of equity securities, and a change in a registrant’s certifying accountant.
A partial listing of a company’s accounts is presented below:
Revenues $80,000
Operating Expenses $50,000
Foreign Currency Translation Adjustment Gain, Net of Tax $4,000
Income tax expense $10,000
What amount should the company report as net income?
A. $20,000 B. $24,000 C. $30,000 D. $34,000
The correct answer is: A
Explanation:
The correct answer is (A).
Revenues $80,000 – Operating expenses $50,000 – Income tax expenses $10,000 = $20,000 Net income.
Foreign currency translation is the process of expressing a foreign entity’s functional currency financial statements in the reporting currency. Translation adjustments are included in the Cumulative Translation Adjustment (CTA) account, which is a component of Other Comprehensive Income (OCI). It is not included with net income.
At what value should a nongovernmental not-for-profit organization record share of stock when received?
A. Donor's basis. B. Average of donor's basis and fair value on date of donation. C. Fair value at end-of-year. D. Fair value on the date of donation.
The correct answer is: D
Explanation:
The correct answer is (D)
All non-cash contributions or gifts-in-kind received by a nongovernmental not-for-profit organization are to be recorded at their fair market values as on the date of the donation. In the given case, the share of stock received by the NPO is an example of gifts-in-kind and is subjected to measurement at the fair value on the date of the donation.
SEC Regulation S-K requires that contracts that are not made in ordinary course of business i.e. material contracts to be filed for:
A.
All SEC reporting entities to file material contracts that were entered into past two years before filing/registration with SEC.
B.
Newly SEC reporting entities to file material contracts that were entered into past two years before filing/registration with SEC.
C.
Newly SEC reporting entities to file material contracts that were entered into past five years before filing/registration with SEC.
D.
All SEC reporting entities to file material contracts that were entered into past five years before filing/registration with SEC.
The correct answer is: B
Explanation:
The correct answer is (B)
Previously all companies are required to file material contracts that were entered within past two years before registration but with applicable changes as per the FAST act Modernization and Simplification of Regulation S-K only newly reporting companies would be required to file material contracts that were entered within past two years before registration/filing with SEC.
Deficits accumulated during the development stage of a company should be
A.
Reported as organization costs
B.
Reported as a part of stockholders’ equity
C.
Capitalized and written off in the first year of principal operations
D.
Capitalized and amortized over a five-year period beginning when principal operations commence
The correct answer is: B
Explanation:
The balance sheet of a development stage enterprise should include any cumulative net losses reported with a caption such as “deficit accumulated during the development stage” in the stockholders’ equity section. It is never acceptable to capitalize a deficit. A deficit is a debit item that belongs in the equity section of the balance sheet, not in the asset section.
At December 31, Grey, Inc., owned 90% of Winn Corp., a consolidated subsidiary, and 20% of Carr Corp., an investee in which Grey cannot exercise
significant influence. On the same date, Grey had receivables of $300,000 from Winn and $200,000 from Carr. In its December 31 consolidated balance sheet,
Grey should report accounts receivable from affiliates of
A. $500,000 B. $340,000 C. $230,000 D. $200,000
The correct answer is: D
Explanation:
Consolidated financial statements should not include intercompany receivables and payables from consolidated subsidiaries. Therefore, the full receivable
from Winn is eliminated in the preparation consolidated financial statements. Since Carr is not a consolidated subsidiary, the full $200,000 receivable from
Carr should be included in the consolidated balance sheet.
Which of the following items is included in the financing activities section of the statement of cash flows?
A.
Cash effects of transactions involving making and collecting loans.
B.
Cash effects of acquiring and disposing of investments and property, plant, and equipment.
C.
Cash effects of transactions obtaining resources from owners and providing them with a return on their investment.
D.
Cash effects of transactions that enter into the determination of net income.
The correct answer is: C
Explanation:
Included in the financing activities section of the statement of cash flows are cash effects of (1) obtaining resources from owners and providing them with a return on their investment, (2) borrowing money and repaying amounts borrowed, or otherwise settling the obligation, (3) obtaining and paying for other resources obtained from creditors on long-term credit, and (4) derivatives that contain a financing component and are accounted for a fair-value or cash-flow hedges. The cash effects of transactions involving making and collecting loans and those of acquiring and disposing of investments and property, plant, and equipment would be included in the investing activities section. The cash effects of transactions and other events that enter into the determination of net income would be included in the operating activities section.
Which of the following assets or transactions is an element of comprehensive income?
A. Investments by owners B. Sales revenue C. Distributions to owners D. Deferred revenue
The correct answer is: B
Explanation:
Comprehensive income includes all changes in equity during a period except those resulting from investments by owners and distributions to owners. Sales revenue would change equity during the period whereas deferred revenue would not.
Options (A), (C) and (D) are incorrect based on the above explanation.
According to the FASB’s conceptual framework, asset valuation accounts are
A. Assets B. Neither assets nor liabilities C. Part of stockholders' equity D. Liabilities
The correct answer is: B
Explanation:
A separate item that reduces or increases the carrying amount of an asset is sometimes found in financial statements. For example, an estimate of uncollectible amounts reduces receivables to the amount expected to be collected, or a premium on a bond receivable increases the receivable to its cost or present value. Those ‘valuation accounts’ are part of the related asset or liability and do not stand on their own.
On January 2, year 3, Pare Co. purchased 75% of Kidd Co.’s outstanding common stock. Selected balance sheet data at December 31, year 3, is as follows:
Pare Kidd
Total assets $420,000 $180,000
Liabilities 120,000 60,000 Common stock 100,000 50,000 Retained earnings 200,000 70,000 $420,000 $180,000 During year 3, Pare and Kidd paid cash dividends of $25,000 and $5,000, respectively, to their shareholders. There were no other intercompany transactions.
In its December 31, year 3, consolidated statement of retained earnings, what amount should Pare report as dividends paid?
A. $ 5,000 B. $25,000 C. $26,250 D. $30,000
The correct answer is: B
Explanation:
The correct answer is (B).
The amount paid by Kidd to Pare (75% × $5,000) is eliminated on a consolidated statement of retained earnings and the 25% amount paid to minority shareholders reduces the Noncontrolling Interest balance and doesn’t affect the statement of retained earnings.
In a consolidated statement of retained earnings will not include dividends paid by Kidd and shall include only Parent’s (Pare’s Dividend) i.e. $25,000.
Palm City uses the modified approach for reporting eligible infrastructure assets. In which of the following components of its basic financial statements, if any, would Palm report this information?
A. Letter of transmittal B. Statement of activities C. Notes to the financial statements D. Not required to report
The correct answer is: C
Explanation:
The correct answer is (C).
Under the modified approach of accounting for infrastructure assets, the assets are not depreciated as they are assumed to have an indefinite life. Palm City would report the use of a modified approach for reporting eligible infrastructure assets in the notes to financial statements.
Notes to financial statements for a government entity should include:
Summary of accounting policies.
Description of reporting entity.
Disclosures of cash and investments, capital assets, long term debt, pensions, commitments, and contingencies.
Information on exceeding the budget at the legal level of control.
Disclosure of individual funds with deficit fund balances.
Risk related to deposit and investments.
Description of the government-wide statements.
Policy for capitalizing fixed assets and estimating useful lives.
Segment information for enterprise funds.
Policy for recording infrastructures.
Disclosures about capital asset impairment.
King, Inc. owns 70% of Simmon Co.’s outstanding common stock. King’s liabilities total $450,000, and Simmon’s liabilities total $200,000. Included in Simmon’s financial statements is a $100,000 note payable to King. What amount of total liabilities should be reported in the consolidated financial statements?
A. $520,000 B. $550,000 C. $590,000 D. $650,000
The correct answer is: B
Explanation:
In consolidated financial statements, liabilities of King’s and Simmon’s are both reported excluding liabilities payable to King. Therefore, total liabilities reported on consolidated Financial Statements would be $550,000.
Ref
Summary
Amount
a
King’s liabilities
$450,000
b
Simon’s liabilities
200,000
c
Notes payable to king
100,000
d
Total liabilities (a+b-c)
$550,000
Options (A) and (C) are incorrect as per the above explanation.
Option (D) is incorrect because $650,000 does not exclude $100,000 liability payable to King in the consolidated financial statements.
The senior accountant for Carlton Co., a public company with a complex capital structure, has just finished preparing Carlton’s income statement for the current fiscal year. While reviewing the income statement, Carlton’s finance director noticed that the earnings per share data has been omitted. What changes will have to be made to Carlton’s income statement as a result of the omission of the earnings per share data?
A.
No changes will have to be made to Carlton’s income statement. The income statement is complete without the earnings per share data.
B.
Carlton’s income statement will have to be revised to include the earnings per share data.
C.
Carlton’s income statement will only have to be revised to include the earnings per share data if Carlton’s market capitalization is greater than $5,000,000.
D.
Carlton’s income statement will only have to be revised to include the earnings per share data if Carlton’s net income for the past two years was greater than $5,000,000.
The correct answer is: B
Explanation:
Publicly-held companies are required to present basic EPS and dilutive EPS on the face of the income statement for:
Income from continuing operations.
Net income.
Option (A), (C) and (D) are incorrect because EPS data is an integral part of the presentation of financial statements for the publicly-held companies.
How should operating expenses for a nongovernmental not-for-profit organization be reported?
A. Change in net assets with donor restrictions. B. Change in net assets without donor restrictions. C. Contra-account to associated revenues. D. None of the above.
The correct answer is: B
Explanation:
Expenses are reported only in the net assets without donor restrictions column. Operating expenses would not be associated with any restrictions and are not reported as a contra-account to associated revenues. Option (a), (c) and (d) are incorrect as per the above explanation.
A donor gives $10,000 to a non-governmental, not-for-profit organization with instructions that it must be used to fund the organization’s general operating expenses during the following fiscal year. The donation will increase the organization’s
A. Unrestricted net assets. B. Restricted net assets. C. Restricted contra assets. D. Restricted retained earnings.
The correct answer is: B
Explanation:
The correct answer is (B).
A donor gives $10,000 to a non-governmental, not-for-profit organization with instructions that it must be used to fund the organization’s general operating expenses during the following year. The donation will increase the organization’s restricted net assets. Restricted net assets are assets of a non-governmental, not-for-profit entity that have special restrictions that are imposed by the donor. Only a donor can set such a restriction. The restriction stipulates how the assets must be used and at what time. The restriction can also be removed after a certain time has passed.
A company’s cash-basis net income for the year ended December 31 was $75,000. The following information is from the company’s accounting records:
January 1
December 31
Accounts receivable
$15,000
$20,000
Prepaid expenses
7,000
4,000
Accrued liabilities
2,500
2,000
What is the accrual-basis net income?
A. $72,500 B. $75,000 C. $77,500 D. $83,500
The correct answer is: C
Explanation:
The correct answer is (C).
On an accrual basis, the company would report net income as on December 31 of $77,500.
Summary
Amount
Net income as per cash basis
$75,000
Increase in accounts receivable
$5,000
Decrease in prepaid expenses
$(3,000)
Decrease in accrued liabilities
$500
Net income as per accrual basis
$77,500
(A) is incorrect because increase in accounts receivable is not considered (i.e. $72,500 = $75,000 - $3,000 + $500).
(B) is incorrect because accrual basis and cash basis net income would not be same. $75,000 is the cash basis net income which should be adjusted for accounts receivable, prepaid expenses and accrued liabilities.
(D) is incorrect because decrease in prepaid expenses should be deducted instead of adding it (i.e. $83,500 = $75,000 + $5,000 + $3,000 + $500).
Accumulated Other Comprehensive Income is reported in which of the following financial statements?
A. The income statement B. The statement of comprehensive income C. The statement of cash flows D. The statement of financial position
The correct answer is: D
Explanation:
The correct answer is (D).
An entity is required to report Accumulated Other Comprehensive Income separately from Retained Earnings, Capital Stock, and Additional Paid-In Capital in the Equity section of the Statement of Financial Position (Balance Sheet).
It is not reported in the Income Statement, Statement of Comprehensive Income, or Statement of Cash Flows.
Accumulated Other Comprehensive Income is a component of equity that includes the total of Other Comprehensive Income for the current period and previous periods. Total Shareholders’ Equity is reported in the Statement of Financial Position.
Which of the following statements regarding dividend distributions to equity method investees, when the distributions are in excess of earnings attributable to investor entities, is true?
A.
The return of the investment would be handled the same as the return on the investment.
B.
There would be no earnings to be adjusted for the cash distribution in the preparation of the statement of cash flows.
C.
The return of investments would need to be reflected in the financing section of the statements.
D.
All of the above are true.
The correct answer is: B
Explanation:
When dividends are distributed by equity method investees, and when the distributions are in excess of earnings attributable to investor entities, the return of the investment would be handled differently than the return on the investment. Essentially, there would be no earnings to be adjusted for the cash distribution in the preparation of the statement of cash flows. The return of investments would need to be reflected in the investing section of the statements.
For an OCBOA presentation, which of the following statements is false regarding the pure cash basis of accounting?
A.
Revenues are recognized when cash is received, rather than when earned.
B.
Expenses are recognized when cash is disbursed rather than when incurred.
C.
Long-term assets are not capitalized, and, hence, no depreciation or amortization is recorded.
D.
Accruals should be made for taxes.
The correct answer is: D
Explanation:
Under the pure cash basis, revenues are recognized when cash is received, rather than when earned; expenses are recognized when cash is disbursed rather than when incurred; long-term assets are not capitalized, and, hence, no depreciation or amortization is recorded; and no accruals are made for taxes and no prepaid expenses are recorded.
A company reported net income available to common stockholders of $2,000,000 for the year ended December 31, year 2. The company had 1,500,000 shares of common stock outstanding as of January 1, year 2, and issued 500,000 additional shares of common stock on May 1, year 2. What amount is the company’s basic earnings per share for the year ended December 31, year 2?
A. $1.00 B. $1.09 C. $1.20 D. $1.33
The correct answer is: B
Explanation:
The correct answer is (B).
Income available to common shareholders = $2,000,000.
Calculation of weighted average number of common shares:-
The common shares issued on May 1 will be appropriated for 8 months: 500,000 x 8 / 12 = 333,333 shares.
Weighted average number of shares outstanding = 1,500,000 + 333,333 = 1,833,333. Basic EPS = $2,000,000 / 1,833,333 = $1.09.
(A) is incorrect because the new issue on May 1 has to be prorated as outstanding for 8 months of the year.
(C) is incorrect is incorrect due to inaccurate calculation.
(D) is incorrect because the new issue on May 1 is ignored in calculating the average number of shares.
When computing diluted earnings per share, convertible securities are
A. Ignored. B. Recognized whether they are dilutive or anti-dilutive. C. Recognized only if they are anti-dilutive. D. Recognized only if they are dilutive.
The correct answer is: D
Explanation:
Convertible securities are only recognized in the computation of diluted earnings per share for any period for which their effect is dilutive.
The IASB Conceptual Framework addresses which of the following concepts of capital?
A. Financial and nonfinancial B. Financial and physical C. Physical and intangible D. Tangible and intangible
The correct answer is: B
Explanation:
The IASB Conceptual Framework addresses the concepts of financial and physical maintenance. Under a financial concept of capital, such as invested money or invested purchasing power, capital is synonymous with the net assets or equity of the entity. Under a physical concept of capital, such as operating capability, capital is regarded as the productive capacity of the entity based on.
Parker Corp. owns 80% of Smith, Inc.’s common stock. During the current year, Parker sold Smith $250,000 of inventory on the same terms as sales made to
third parties. Smith sold all of the inventory purchased from Parker in this year. The following information pertains to Smith and Parker’s sales for the year:
Parker Smith Sales $1,000,000 $ 700,000 Cost of sales (400,000) (350,000) $ 600,000 $ 350,000 What amount should Parker report as cost of sales in its year-end consolidated income statement?
A. $750,000 B. $680,000 C. $500,000 D. $430,000
The correct answer is: C
Explanation:
Parker, in recording the inventory sale to its subsidiary, Smith, recognized cost of goods sold of $100,000 [i.e., $250,000 × ($400,000 / $1,000,000)]. In
recording the later sale of the same inventory to an unrelated customer, Smith recognized cost of goods sold of $250,000. However, from a consolidated
perspective, the sale to Smith (the affiliated company) did not occur. Therefore, the amount to be reported as cost of goods sold in the consolidated income
statement is $500,000 (i.e., $400,000 + $350,000 - $250,000).
Cancer Educators, a not-for-profit organization, incurred costs of $10,000 when it combined program functions with fundraising functions. Which of the
following cost allocations might Cancer report in its statement of activities?
Program Services Fund Raising General Services A $0 $0 $10,000 B $0 $6,000 $ 4,000 C $ 6,000 $4,000 $0 D $10,000 $0 $0
The correct answer is: C
Explanation:
All joint costs of informational materials or activities should be reported as fundraising expenses unless it can be demonstrated that a program or management
and general function has been conducted in conjunction with the appeal for funds. In the question, it appears that a program function and a fund-raising appeal
have joint costs. There is the alternative of no cost allocation and classifying all costs as fundraising costs, but this is not one of the options. Answers (a) and
(b) are not appropriate; no general service is mentioned as having been accomplished. Since no general services are mentioned, no allocation would be
appropriate to this function. Only answer (c) which allows for joint allocation might be appropriate. One must assume that the 60/40 allocation is appropriate
under the circumstances - although no information is given to verify this assumption.
During the current year, both Raim Co. and Cane Co. suffered losses due to the flooding of the Mississippi River. Raim is located two miles from the river and
sustains flood losses every two to three years. Cane, which has been located fifty miles from the river for the past twenty years, has never before had flood
losses. How should the flood losses be reported in each company’s year-end income statement?
Raim Cane
A As a component of income from continuing operations As a component of income from continuing operations or in the Financial Statement Notes
B As as an extraordinary item, net of tax As a component of income from continuing operations
C As an extraordinary item, pre-tax As a component of income from continuing operations
D As a component of income from continuing operations or in the Financial Statement Notes As a component of income from continuing operations or in the Financial Statement Notes
The correct answer is: A
Explanation:
For unusual or infrequent Items, there are two options for reporting:
Income Statement (above Income from Continuing Operations)
Footnotes to Financial Statements
Unusual or infrequent Items get Retrospective or Prospective treatment in the financial statements.
(A) is correct because Raim sustains these losses on a regular basis, so they are not unusual or infrequent and should be shown as a component of income from continuing operations.
Cane’s loss is unusual/infrequent, so Cain can present the loss using one of the two methods listed above.
The purpose of a statement of financial position for a nongovernmental not-for-profit entity is to provide relevant information about
A.
The cash receipts and cash payments during a period in time
B.
The effects of transactions and other events and circumstances that change the amount and nature of net assets
C.
The assets, liabilities, net assets and about their relationships to one another at a moment in time
D.
The changes in net assets with donor restrictions and net assets without donor restrictions for a period of time
The correct answer is: C
Explanation:
The correct answer is (C).
Purpose of a statement of financial position for a non-governmental, not-for-profit entity is to provide relevant information about the assets, liabilities, net assets, and about their relationships to one another at a moment in time.
(A) is incorrect because the statement of cash flows provides information about the cash receipts and cash payments during a period in time.
(B) and (D) are incorrect because such information is provided in the statement of activities.
The SEC system that is intended to benefit electronic filers, enhance the speed and efficiency of SEC process, and readily make financial information available to users is commonly referred to as what?
A. EDGAR B. EDGRE C. EDF D. ETF
The correct answer is: A
Explanation:
EDGAR stands for the Electronic Data Gathering, Analysis and Retrieval System. EDGAR is intended to benefit electronic filers, enhance the speed and efficiency of SEC process, and readily make financial information available to users.
Inflation makes which of the following assumptions questionable?
A. Going concern B. Economic entity C. Unit-of-measure D. None of the above
The correct answer is: C
Explanation:
The unit-of-measure assumption presupposes that costs incurred at different points in time are intermingled in the accounts, and thus, it must be assumed that the purchasing power of the dollar remains constant over time. Inflation makes this assumption questionable.
Which of the following should be disclosed in the summary of significant accounting policies?
Composition of inventories Maturity dates of long-term debt A Yes Yes B Yes No C No No D No Yes
The correct answer is: C
Explanation:
The summary of significant accounting policies should identify and describe the accounting principles followed by the reporting entity and the methods of applying those principles. Examples of disclosures by a business enterprise commonly required with respect to accounting policies include those relating to basis of consolidation, depreciation methods, amortization of intangibles, inventory pricing, and recognition of profit on long-term construction-type contracts. Financial statement disclosure of accounting policies should not duplicate details (for example, composition of inventories or maturity dates of long-term debt) presented elsewhere as part of the financial statements.
A storm damaged the roof of a new building owned by K-9 Shelters, a not-for-profit organization. A supporter of K-9, a professional roofer, repaired the roof at no charge. In K-9’s statement of activities, the damage and repair of the roof should
A. Be reported by note disclosure only. B. Be reported as an increase in both expenses and contributions. C. Be reported as an increase in both net assets and contributions. D. Not be reported.
The correct answer is: B
Explanation:
Donated services are recognized as revenues if nonfinancial assets are created or enhanced, special skills are required that would otherwise be purchased, the value is measurable, and the entity controls the employment and duties of the service donors (similar to an employer-employee relationship). The work performed by the roofer meets these criteria and as such would be reported as an increase in both expenses and contributions.
An entity’s cash purchases and sales of investments considered as cash equivalents are generally part of the entity’s __________ activities, and details of those transactions need not be reported in a statement of cash flows.
A. Investing B. Operating C. Cash management D. Financing
The correct answer is: C
Explanation:
An entity’s cash purchases and sales of investments considered as cash equivalents generally are part of the enterprise’s cash management activities, and the details of those transactions need not be reported in a statement of cash flows.
Beni Corp. purchased 100% of Carr Corp.’s outstanding capital stock for $430,000 cash. Immediately before the purchase, the balance sheets of both
corporations reported the following:
Beni Carr
Assets $2,000,000 $750,000
Liabilities 750,000 400,000
Common stock 1,000,000 310,000
Retained earnings 250,000 40,000
Liabilities and stockholders’ equity $2,000,000 $750,000
At the date of purchase, the fair value of Carr’s assets was $50,000 more than the aggregate carrying amounts. In the consolidated balance sheet prepared
immediately after the purchase, the consolidated stockholders’ equity should amount to
A. $1,680,000 B. $1,650,000 C. $1,600,000 D. $1,250,000
The correct answer is: D
Explanation:
The amount of consolidated stockholders’ equity at the date of purchase is equal to Beni’s stockholders’ equity of $1,250,000. The subsidiary’s stockholders’
equity accounts are eliminated through consolidation so that only the asset and liability accounts of the subsidiary remain to be combined with the parent
company accounts.
A company records items on the cash basis throughout the year and converts to an accrual basis for year-end reporting. Its cash-basis net income for the year is $70,000. The company has gathered the following comparative balance sheet information:
Beginning of year End of year Accounts payable $ 3,000 $1,000 Unearned revenue 300 500 Wages payable 300 400 Prepaid rent 1,200 1,500 Accounts receivable 1,400 600 What amount should the company report as its accrual-based net income for the current year?
A. $68,800 B. $70,200 C. $71,200 D. $73,200
The correct answer is: C
Explanation:
In cash-basis accounting, the effects of transactions and other events on the assets and liabilities of a business enterprise are recognized and reported only when cash is received or paid; while in accrual accounting, these effects are recognized and reported in the time periods to which they relate. Cash-basis accounting does not attempt to match revenues and the expenses associated with those revenues. If liabilities have a net decrease, then cash is assumed to have been used, and cash net income would be lower than accrual. The same logic holds true for the asset side. If current assets increase, cash was consumed, so cash net income is less than accrual. A short-cut method is to journalize the net change of each account, and plug the difference to cash, as follows:
Accounts payable 2,000 Prepaid Rent 300 Unearned revenue 200 Wages payable 100 Accounts receivable 800 Cash (plug) 1,200 Overall, net cash decreased by $1,200, so Cash Net Income is $1,200 less than Accrual Net Income; Accrual Net Income was $71,200.
Mend Co. purchased a three-month U.S. Treasury bill. Mend’s policy is to treat as cash equivalents all highly liquid investments with an original maturity of three months or less when purchased. How should this purchase be reported in Mend’s statement of cash flows?
A. As an outflow from operating activities B. As an outflow from investing activities C. As an outflow from financing activities D. Not reported
The correct answer is: D
Explanation:
In the Statement of Cash Flows, the change in cash and cash equivalents is measured. Cash equivalents are short- term highly liquid investments that have both of the following characteristics: -Readily convertible to cash (highly liquid) and So near to their maturity that they present insignificant risk of changes in value because of changes in interest rates. Generally, only investments with an original maturity of 3 months or less from the purchase date qualify to be cash equivalents.In the given scenario, Mend Company’s purchase of three-month US Treasury bill will ideally be not reported since it is an exchange of cash with a cash equivalent and as such there is no inflows or outflows of cash.Option (a) is in correct because operating activities includes inflows and outflows of cash related to the production of income from continuing operations, which is not the case here as it’s an exchange of cash with a cash equivalent.Option (b) is incorrect because investing activities includes inflows and outflows of cash generally related to long term investments or non-current assets.Option (c) is incorrect because financing activities includes inflows and outflows of cash related to debt or equity.
Flax Corp. uses the direct method to prepare its statement of cash flows. Flax’s trial balances at December 31, year 2 and year 1, are as follows:
December 31 Year 2 Year 1 Debits: Cash $35,000 $32,000 Accounts receivable 33,000 30,000 Inventory 31,000 47,000 Property, plant, and equipment 100,000 95,000 Unamortized bond discount 4,500 5,000 Cost of goods sold 250,000 380,000 Selling expenses 141,500 172,000 General and administrative expenses 137,000 151,300 Interest expense 4,300 2,600 Income tax expense 20,400 61,200 $756,700 $976,100
Year 2 Year 1 Credits: Allowance for uncollectible accounts $1,300 $1,100 Accumulated depreciation 16,500 15,000 Trade accounts payable 25,000 17,500 Income taxes payable 21,000 27,100 Deferred income taxes 5,300 4,600 8% callable bonds payable 45,000 20,000 Common stock 50,000 40,000 Additional paid-in capital 9,100 7,500 Retained earnings 44,700 64,600 Sales 538,800 778,700 $756,700 $976,100 Flax purchased $5,000 in equipment during year 2. Flax allocated one-third of its depreciation expense to selling expenses and the remainder to general and administrative expenses. What amounts should Flax report in its statement of cash flows for the year ended December 31, year 2, for cash paid for selling expenses?
A. $142,000 B. $141,500 C. $141,000 D. $140,000
The correct answer is: C
Explanation:
Selling expenses $141,500
Less allocated portion of depreciation expense
[($16,500 - $15,000) / 3] ($500)
Cash paid for selling expenses $141,000
*Accumulated Depreciation:
Year 2: $16,500
Year 1: $15,000
We can calculate Depreciation for Year 2 = $1,500 i.e. Increase in Accumulated Depreciation ($16,500 - $15,000).
As the question mentions one-third of depreciation expense is allocated to selling expenses.
Total Depreciation in Selling Expenses = $1,500/3 = $500.
Rock Co.’s financial statements had the following balances at December 31:
Ordinary gain $ 50,000
Foreign currency translation gain 100,000
Net Income 400,000
Unrealized gain on available-for-sale debt securities 20,000
What amount should Rock report as comprehensive income for the year ended December 31?
A. $400,000 B. $420,000 C. $520,000 D. $570,000
The correct answer is: C
Explanation:
Comprehensive income includes all changes in equity during a period except those resulting from investments by owners and distributions to owners. Comprehensive income is divided into net income and other comprehensive income. An entity must classify items of other comprehensive income by their nature, in one of these classifications: foreign currency items, minimum pension liability adjustments, unrealized gains and losses on certain investments in debt securities, and gains and losses on cash flow hedging derivative instruments. All regular items of income and expense, including unusual and infrequently occurring items, are included in the determination of net income. The ordinary gain is already included in the net income amount that is given in this problem. Translation adjustments and unrealized holding gains and losses are reported in other comprehensive income (OCI). They are excluded from current earnings and should not be included in the determination of net income.
Clark Co. had the following transactions with affiliated parties during the current year:
Sales of $60,000 to Dean, Inc. with $20,000 gross profit. Dean had $15,000 of this inventory on hand at year end. Clark owns a 15% interest in Dean and does not exert significant influence.
Purchases of raw materials totaling $240,000 from Kent Corp., a wholly-owned subsidiary. Kent’s gross profit on the sale was $48,000. Clark had $60,000 of this inventory remaining on December 31.
Before eliminating entries, Clark had consolidated current assets of $320,000. What amount should Clark report in its December 31 consolidated balance
sheet for current assets?
A. $320,000 B. $317,000 C. $308,000 D. $303,000
The correct answer is: C
Explanation:
When one company sells merchandise to an affiliate at a price above cost, the ending inventory of the buyer contains an element of unrealized gross profit.
The gross profit is not realized to the economic entity until the inventory is sold to an unaffiliated company. The preparation of the consolidated financial
statements requires that the unrealized gross profit is eliminated from inventory. Dean is not an affiliate of Clark because Clark cannot exercise significant
influence over Dean year-end its investment (i.e., Clark only owns a 15% interest in Dean). Therefore, no elimination entry should be made for the
transaction with Dean.
Consolidated current assets before elimination entries $320,000
Less: Unrealized gross profit on intercompany
inventory transfer to wholly-owned subsidiary
[($60,000 / $240,000) × $48,000] (12,000)
Consolidated current assets to be reported in
consolidated balance sheet $308,000
Which of the following disclosures should prospective financial statements include?
Summary of significant accounting policies Summary of significant assumptions A Yes Yes B Yes No C No Yes D No No
The correct answer is: A
Explanation:
Disclosures for prospective financial statements should include summaries of both significant accounting policies and significant assumptions.
The Jackson Foundation, a not-for-profit organization, received contributions during the year as follows:
Unrestricted cash contributions of $500,000.
Cash contributions of $200,000 to be restricted to acquisition of property.
Jackson’s statement of cash flows should include which of the following amounts?
Operating activities Investing activities Financing activities
A $700,000 $0 $0
B $500,000 $200,000 $0
C $500,000 $0 $200,000
D $0 $500,000 $200,000
The correct answer is: C
Explanation:
Unrestricted cash from contributors is included in operating activities. Investing activities include the sale and purchase of investments and PP&E. With nonprofit orgfanizations, the description of cash flows from financing activities expands to include donor-restricted cash that must be used for long-term purposes.
Which of the following Acts requires the filing of periodic reports with the SEC?
A. Securities Act of 1933 B. Securities Exchange Act of 1934 C. Trust Indenture Act of 1939 D. Public Company Reform and Investor Protection Act of 2002
The correct answer is: B
Explanation:
Once companies registered, the Securities Exchange Act of 1934 requires that companies file periodic reports to update financial information on their companies. The most common Exchange Act reports include Form 10-K (annual reports), Form 10-Q (quarterly reports) and Form 8-K (current reports).
Terra Co.’s total revenues from its three operating segments were as follows:
Segment Sales to External Customers Intersegment Sales Total Revenues Lion $ 70,000 $30,000 $100,000 Monk 22,000 4,000 26,000 Nevi 8,000 16,000 24,000 Combined $100,000 50,000 150,000 Elimination \_\_\_\_\_\_ (50,000) (50,000) Consolidation $100,000 $---- $100,000Which operating segment(s) is(are) deemed to be reportable segments? A. None. B. Lion only. C. Lion and Monk only. D. Lion, Monk, and Nevi.
The correct answer is: D
Explanation:
An operating segment is deemed to be a reportable segment if it meets one or more of the revenue, profit(loss), and assets tests. If an operating segment’s revenue, including both sales to external customers and intersegment sales and transfers, is 10% or more of the combined revenue of all operating segments, it meets the revenue test. Lion, Monk, and Nevi each have more than 10% of combined revenues and are deemed to be reportable segments.
Which of the following describes GASB’s role for improving communications (as described in Concept Statement No. 3) in governmental external financial reporting?
A.
The Board’s independence supports governments’ credibility and comparability among governments’ financial statements.
B.
GASB standards establish performance standards that users can use to evaluate the effectiveness of governmental services.
C.
Developing key financial measures for governmental service performance.
D.
Representing state and local governments at Congressional oversight hearings with the Government Accountability Office.
The correct answer is: A
Explanation:
Concepts Statement No. 3 describes the importance of the GASB’s independence for establishing communication principles in the reporting standards to support governments’ credibility and enhance comparability among external financial statements.
On January 2, year 3, Pare Co. purchased 75% of Kidd Co.’s outstanding common stock. Selected balance sheet data at December 31, year 3, is as follows:
Pare Kidd Total assets $420,000 $180,000 Liabilities 120,000 60,000 Common stock 100,000 50,000 Retained earnings 200,000 70,000 $420,000 $180,000 During year 3, Pare and Kidd paid cash dividends of $25,000 and $5,000, respectively, to their shareholders. There were no other intercompany transactions. In Pare's December 31, year 3, consolidated balance sheet, what amount should be reported as noncontrolling interest in net assets?
A. $0 B. $30,000 C. $45,000 D. $105,000
The correct answer is: B
Explanation:
Without any specific information provided about amounts on the acquisition date and reported net income you must infer the noncontrolling interest is worth 25% of Kidd’s net assets. Kidd’s net assets are $180,000 – $60,000 = $120,000. $120,000 × 25% = $30,000.
Comprehensive Income excludes changes in equity resulting from which of the following?
A. Loss from discontinued operations B. Prior period error correction C. Dividends paid to stockholders D. Unrealized loss on investments in non-current marketable equity securities
The correct answer is: C
Explanation:
Comprehensive income includes all changes in equity except those caused by owner investments or distributions to owners. Loss from discontinued operations is included in net income (a component of comprehensive income). Prior period error correction is a change in stockholders’ equity. Unrealized loss on investments in non-current marketable equity securities is a change in stockholders’ equity.
Neely Co. disclosed in the notes to its financial statements that a significant number of its unsecured trade account receivables are with companies that operate in the same industry. This disclosure is required to inform financial statement users of the existence of :
A. Concentration of credit risk. B. Concentration of market risk. C. Risk of measurement uncertainty. D. Off-balance sheet risk of accounting loss.
The correct answer is: A
Explanation:
Credit risk is the possibility that a loss may occur from the failure of another party to perform according to the terms of a contract. An entity must disclose all concentrations of credit risk arising from all financial instruments. Group concentrations of credit risk exist if a number of counter parties are engaged in similar activities and have similar economic characteristics that would affect their ability to meet contractual obligations affected by changes in economic or other conditions. Market risk is the possibility that future changes in market prices may make a financial instrument less valuable or more onerous . An entity is encouraged,but not required,to disclose quantitative information about the market risks of financial instruments. There is no note disclosure for risk of measurement uncertainty here. Accounts receivable uses an allowance for doubtful accounts to account for uncertainty of collection. There is no off-balance sheet risk of accounting loss here. Off-balance sheet risk is the risk of accounting loss from a financial instrument that exceeds the amount recognized for the instrument in the balance sheet.
A company is required to file quarterly financial statements with the United States Securities and Exchange Commission on Form 10-Q. The company operates in an industry that is not subject to seasonal fluctuations that could have a significant impact on its financial condition. In addition to the most recent quarter end, for which of the following periods is the company required to present balance sheets on Form 10-Q?
A.
The end of the corresponding fiscal quarter of the preceding fiscal year.
B.
The end of the preceding fiscal year and the end of the corresponding fiscal quarter of the preceding fiscal year.
C.
The end of preceding fiscal year.
D.
The end of the preceding fiscal year and the end of the prior two fiscal years.
The correct answer is: C
Explanation:
The Form 10-Q is a report of a public company’s performance that must be filed quarterly with the SEC. It includes unaudited financial statements and provides a continuing view of the company’s financial position during the year. In addition to the most recent quarter end balance sheet, a company is required to present the balance sheet for the end of the preceding fiscal year on the Form 10-Q..
The following information is relevant to the computation of Chan Co.’s earnings per share to be disclosed on Chan’s income statement for the year ending December 31:
Net income for the year is $600,000.
$5,000,000 face value 10-year convertible bonds outstanding on January 1. The bonds were issued four years ago at a discount being amortized in the amount of $20,000 per year. The stated rate of interest on the bonds is 9%, and the bonds were issued to yield 10%. Each $1,000 bond is convertible into 20 shares of Chan’s common stock.
Chan’s corporate income tax rate is 25%.
Chan has no preferred stock outstanding, and no other convertible securities.
What amount should be used as the numerator in the fraction used to compute Chan’s diluted earnings per share assuming that the bonds are dilutive securities?
A. $130,000 B. $247,500 C. $952,500 D. $1,070,000
The correct answer is: C
Explanation:
For convertible securities, use “If-converted method”- the calculation of diluted earnings per share (EPS) (assume anyone who “could convert” does so) for a company with convertible preferred stock or convertible bonds starts with basic EPS. For numerator, earnings are increased by the dividends or after-tax interest expense that would not have been due if the securities were converted to common stock at beginning of the year, which would be $952,500 ($352,500 + $600,000).
Ref Summary Amount
a Convertible bonds outstanding $5,000,000
b Stated interest rate 9%
c Annual Interest paid on bonds (a x b) $450,000
d Amortization of discount on bonds $20,000
e Interest expense (c+d) $470,000
f Corporate income tax rate 25%
g After tax interest expense {e x (1-0.25)} $352,500
h Net income $600,000
I Numerator for dilutive EPS (g+h) $952,500
Option (A) is incorrect because the interest expense is deducted from net income, instead of adding it to NI and after-tax expense impact is not considered. [$130,000 = $600,000 - $470,000].
Option (B) is incorrect because the interest expense is deducted from net income, instead of adding it to NI. {$247,500 = $600,000 - [$470,000 x (1- 25%)]}.
Option (D) is incorrect because the interest expense should be added after tax to NI. [$1,070,000 = $600,000 + $470,000.
Pyle Hospital received a $250,000 pure endowment fund grant. Pyle’s governing board designated, for special uses, $300,000 which had originated from unrestricted gifts. What amount of these resources should be accounted for as part of unrestricted assets?
A. $0 B. $250,000 C. $300,000 D. $550,000
The correct answer is: C
Explanation:
Unrestricted resources may be appropriated or designated by the governing board for special uses. The board nevertheless has the authority to rescind such actions. Therefore, board-designated assets should be accounted for as part of unrestricted assets. The pure endowment grant is accounted within restricted assets.
Guidance for which of the following is included in the FASB Accounting Standards Codification?
A. Governmental Accounting Standards (GAS) B. Regulatory Accounting Principles (RAP) C. Securities and Exchange Commission (SEC) content D. Other Comprehensive Basis of Accounting (OCBOA)
The correct answer is: C
Explanation:
The Codification includes Securities and Exchange Commission (SEC) content for reference by public companies. It does not include guidance for Other Comprehensive Basis of Accounting (OCBOA), Regulatory Accounting Principles (RAP) or governmental accounting standards.
In a statement of cash flows, which of the following items is reported as a cash outflow from financing activities?
Payments to retire mortgage notes Interest payments on mortgage notes Dividend payments A. I, II, and III B. II and III C. I only D. I and III
The correct answer is: D
Explanation:
Cash flows from financing activities include issuing debt or equity.
Proceeds from issuing or payments for retiring bonds.
Issuance or re-acquisition of stock or treasury stock.
Borrowing or repaying of loan.
Dividends paid to shareholders.
Bank overdrafts which are excluded from cash.
Payments to retire mortgage notes and dividend payments will be considered as out flow under financing activities.Interest payments on mortgage notes will be considered as outflow from operating activities.Options (a), (b) & (c) are incorrect based on the above explanation.
Under IFRS 1, a first-time adopter must do which of the following?
A.
Publish interim financial reports in advance of its first IFRS financial statements
B.
Apply IAS 8 to the changes in accounting policies it makes when it adopts IFRS
C.
Provide supplementary explanations necessary for understanding the transition to IFRS
D.
Disregard the use of deemed cost for an investment in any related entities
The correct answer is: C
Explanation:
In the year of transition to IFRS, an entity must present provide supplementary explanations necessary for understanding the transition to IFRS. IFRS 1 does not require a first-time adopter to publish interim financial reports in advance of an entity’s first IFRS financial statements. IAS 8 does not apply to the changes in accounting policies an entity makes when it adopts IFRS or to changes in those policies until after it presents its first IFRS financial statements. Also, if the first-time adopter uses a deemed cost in its opening IFRS statement of financial position in its separate financial statements for an investment in a subsidiary, associate, or jointly controlled entity, those separate financial statements are required to disclose the aggregate deemed cost of those investments.
Which of the following types of statements is not required when first reporting under IFRS 1?
A. Statement of Changes in Equity B. Statement of Cash Flows C. Statement of Retained Earnings D. Statement of Profit or Loss and Other Comprehensive Income
The correct answer is: C
Explanation:
An entity’s first IFRS financial statements shall include at least three statements of financial position, two statements of profit or loss and other comprehensive income, two separate statements of profit or loss (if presented), two statements of cash flows, and two statements of changes in equity and related notes. The Statement of Retained Earnings is only a US GAAP statement.
Upon first-time adoption of IFRS, an entity may elect to use fair value as deemed cost for:
A.
Biological assets related to agricultural activity for which there is no active market.
B.
Intangible assets for which there is no active market.
C.
Any individual item of property, plant and equipment.
D.
Financial liabilities that are not held for trading.
The correct answer is: C
Explanation:
Assets carried at cost (e.g., property, plant, and equipment) may be carried at their fair value at the opening IFRS balance sheet date. Fair value becomes the “deemed cost” going forward under the IFRS cost model.
Which of the following is a true statement regarding the accounting and reporting requirements related to inventory under both IFRS and U.S. GAAP?
A.
Both IFRS and U.S. GAAP are based on the principle that the primary basis of accounting for inventory is fair value.
B.
Under both IFRS and U.S. GAAP, the cost of inventory includes all direct expenditures to ready inventory for sale, including selling and storage costs.
C.
Under both IFRS and U.S. GAAP, inventory is carried at the lower of cost market, and any write-downs of inventory create a new cost basis that subsequently cannot be reversed.
D.
Under IFRS, LIFO is prohibited.
The correct answer is: D
Explanation:
Under IFRS, LIFO is prohibited; under U.S. GAAP it is an acceptable costing method. Under both U.S. GAAP and IFRS, the primary basis of accounting for inventory is cost. The cost of inventory includes all direct expenditures to ready inventory for sale, but excludes selling and storage costs. Inventory is carried at the lower of cost or market with any write-downs of inventory to the lower of cost or market creating a new cost basis; however, under IFRS, previously recognized impairment losses can be reversed, up to the amount of the original impairment loss, when the reasons for the impairment no longer exist.
On August 1, Year 2, Lex Co. decided to adopt IFRS. The company’s first IFRS reporting period is for the year ended December 31, Year 2. Lex will present Year 1 statements for comparative purposes. What is Lex’s date of transition to IFRS?
A. January 1, Year 1 B. January 1, Year 2 C. August 1, Year 2 D. August 1, Year 1
The correct answer is: A
Explanation:
The date of transition is the beginning of the earliest period for which comparative information is presented, which is January 1, Year 1.
King Corp. owns 80% of Lee Corp.’s common stock. During October of the current year, Lee sold merchandise to King for $100,000. At December 31, one-half
of the merchandise remained in King’s inventory. For the year, gross profit percentages were 30% for King and 40% for Lee. The amount of
intercompany profit in ending inventory at December 31 that should be eliminated in consolidation is
A. $40,000 B. $20,000 C. $16,000 D. $15,000
The correct answer is: B
Explanation:
When one company sells merchandise to an affiliate at a price above cost, the ending inventory of the buyer contains an element of unrealized gross profit.
The gross profit is not realized to the economic entity until the inventory is sold to an unaffiliated company. The unrealized gross profit in inventory must be
eliminated in the preparation of consolidated financial statements. The amount of unrealized intercompany profit in ending inventory that should be
eliminated for the consolidation in question is determined as follows:
King’s 12/31 inventory acquired from Lee ($100,000 × 50%) $50,000
Lee’s gross profit percentage × 40%
Unrealized intercompany profit in King’s 12/31 inventory $20,000
On December 31 of the current year, the Board of Directors of Maxx Manufacturing, Inc. committed to a plan to discontinue the operations of its Alpha division in the following year. Maxx estimated that Alpha’s following year operating loss would be $500,000 and that Alpha’s facilities would be sold for $300,000 less than their carrying amounts. Alpha’s current year operating loss was $1,400,000 before any consideration of impairment loss. Maxx’s effective tax rate is 30%. These estimates were accurate.
In its following year income statement, what amount should Maxx report as loss from discontinued operations?
A. $350,000 B. $500,000 C. $560,000 D. $800,000
The correct answer is: A
Explanation:
The income statement of a business enterprise for current and prior periods shall report the results of operations of the component in discontinued operations in the period(s) in which they occur.
Alpha’s following year operating loss $500,000
Applicable income tax benefit ($500,000 × 30%) (150,000)
Loss on disposal, net of income taxes $350,000
Based on the stock transactions below, what is the weighted average number of shares outstanding as of March 1, year 2, that should be used in the calculation of basic earnings per share in financial statements issued on March 1, year 2?
Date Transactions January 1, year 1 Beginning balance 100,000 April 1, year 1 Issued 30,000 shares for cash June 1, year 1 50% stock dividend February 15, year 2 2 for 1 stock split March 15, year 2 Issued 40,000 shares for cash A. 147,500 B. 183,750 C. 295,000 D. 367,500
The correct answer is: D
Explanation:
The correct answer is (D)
Basic EPS = Income available to common shareholders/Weighted average number of common shares outstanding.
The shares sold must be prorated for the portion of the year they were outstanding. Since the financial statements for Year 1 are being issued on March 1 Year 2, after the 2 for 1 stock split on February 15 of Year 2, the split will be treated as if it had occurred at the beginning of the earliest period presented which is Year 1. In computing WACS, retroactive application is given to stock splits and stock dividends. WACS is computed as follows:
Common Stock Outstanding Total Number Weighted Shares O/S Shares Outstanding
Beginning balance year = 100,000 100,000
Issued 30,000 shares for cash, Apr 1, Year 1 = (30,000) x (9/12) 22,500
Shares outstanding before stock split 122,500
50% Stock dividend, Jun 1, Year 1 = 122,500 x 150% 183,750
2 for 1 stock split Feb 15, Year 2 = (183,750) x 2 367,500
Which of the following is not included in the FASB Accounting Standards Codification?
A. FASB Statements B. FASB Technical Bulletins C. FASB Interpretations D. FASB Statements of Financial Accounting Concepts
The correct answer is: D
Explanation:
FASB Statements of Financial Accounting Concepts are not part of the FASB Accounting Standards Codification. FASB Statements, Technical Bulletins, and Interpretations are part of the FASB Accounting Standards Codification.
Grid Corp. acquired some of its own common shares at a price greater than both their par value and original issue price but less than their book value. Grid
uses the cost method of accounting for treasury stock. What is the impact of this acquisition on total stockholders’ equity and the book value per common
share?
Total stockholders' equity Book value per share A Increase Increase B Increase Decrease C Decrease Increase D Decrease Decrease
The correct answer is: C
Explanation:
When treasury stock is acquired, total stockholders’ equity decreases by the cost of the treasury shares, regardless of the method used to account for the
treasury stock. The book value per common share is computed by dividing total stockholders’ equity applicable to common stock by the number of common
stock shares outstanding. The acquisition of treasury shares at a price less than their book value will reduce both the numerator and denominator of the book
value ratio; however, the reduction of the numerator is less than the amount that was in there for these shares. Therefore, the excess book value for those
shares would now be spread over the remaining common shares outstanding, resulting in an increase in the book value per common share.
In year 2, the Nord Association, a nongovernmental not-for-profit organization, received a $100,000 contribution to fund scholarships for medical students.
The donor stipulated that only the interest earned on the contribution be used for the scholarships. Interest earned in year 2 of $15,000 will be used to award scholarships in year 3.
What amount should Nord report as net assets with donor restrictions at the end of year 2?
A. $15,000 B. $100,000 C. $ 115,000 D. $0
The correct answer is: C
Explanation:
The correct answer is (C).
Entities are required to classify net assets based upon the existence or absence of donor-imposed restrictions.
Net assets are classified into two categories: Net Assets with donor restrictions and Net Assets without donor restrictions.
Contributions with donor-imposed restrictions are reported as restricted support.
Restricted support increases Net Assets with donor restrictions.
Note: The $100,000 received as a contribution to fund scholarships for medical students is permanently restricted and will be classified as Net Assets with Donor Restrictions, whereas the $15,000 from interest is also temporarily restricted to award scholarships and will be classified as Net Assets with Donor Restrictions.
When the amount is expended to award scholarships, then it is reclassified from Net Assets with Donor Restrictions to Net Assets without Donor Restrictions
Wood Co.’s dividends on noncumulative preferred stock have been declared but not paid. Wood has not declared or paid dividends on its cumulative preferred stock in the current or the prior year and has reported a net loss in the current year. For the purpose of computing basic earnings per share, how should the income available to common stockholders be calculated?
A.
The current-year dividends and the dividends in arrears on the cumulative preferred stock should be added to the net loss, but the dividends on the noncumulative preferred stock should not be included in the calculation.
B.
The dividends on the noncumulative preferred stock should be added to the net loss, but the current-year dividends and the dividends in arrears on the cumulative preferred stock should not be included in the calculation.
C.
The dividends on the noncumulative preferred stock and the current-year dividends on the cumulative preferred stock should be added to the net loss.
D.
Neither the dividends on the noncumulative preferred stock nor the current-year dividends and the dividends in arrears on cumulative preferred stock should be included in the calculation.
The correct answer is: C
Explanation:
The correct answer is (C).
A preferred dividend on the cumulative preferred stock is deducted from the net income or added to the net loss regardless of the fact it’s declared or not.
The dividends on the non-cumulative preferred stock have been declared but not paid. So, the dividends on the noncumulative preferred stock should be added to the net loss.
Therefore, the current year dividends on the cumulative preferred stock (current year only) and noncumulative preferred stock should be added to the net loss.
(A) is incorrect because dividends in arrears for cumulative preferred shares should not be added to the net loss when not declared. The dividends on noncumulative shares should be included when declared in a net loss.
(B) is incorrect because the current year dividends for cumulative preferred shares are added to a net loss.
(D) is incorrect because the current year dividends should be included in net loss for non-cumulative preference shares and for cumulative preference shares
On January 1 of the current year, Poe Corp. sold a machine for $900,000 to Saxe Corp., its wholly owned subsidiary. Poe paid $1,100,000 for this machine,
which had accumulated depreciation of $250,000. Poe estimated a $100,000 salvage value and depreciated the machine on the straight-line method over 20
years, a policy which Saxe continued. In Poe’s December 31 consolidated balance sheet, this machine should be included in cost as
A. $1,100,000 B. $1,000,000 C. $ 900,000 D. $ 850,000
The correct answer is: A
Explanation:
In the consolidated balance sheet, the machine’s cost must be based upon the original cost of the machine to the initial buyer. Therefore, the machine is
reported at its cost to the consolidated entity of $1,100,000.
According to the FASB conceptual framework, the objective of general purpose financial reporting is based on
A. Generally accepted accounting principles B. Reporting on management's stewardship C. The need for conservatism D. The needs of the users of the information
The correct answer is: D
Explanation:
The primary function of accounting is to provide quantitative useful information, based on the needs of the users. The objective of general purpose financial reporting for business enterprises is not based on generally accepted accounting principles, reporting on management’s stewardship, or the need for conservatism.
Under Abbey Hospital’s established rate structure, the hospital would have earned patient service revenue of $6,000,000 for the year. However, Abbey did not expect to collect this amount because of charity care of $1,000,000 and discounts of $500,000 to third-party payers. How much should Abbey record as patient service revenue for the year?
A. $6,000,000 B. $5,500,000 C. $5,000,000 D. $4,500,000
The correct answer is: C
Explanation:
Abbey’s patient service is determined by subtracting the charity care from the patient service revenue that would have been recorded at Abbey’s established rate for all health care services provided (i.e.,$6,000,000 - $1,000,000 = $5,000,000). Charity care is not included in patient service revenues because these services were provided free of charge and, thus, were never expected to result in cash flows. On the other hand, the discounts to third-party payors are reported as deductions from patient service revenues to determine net patient service revenue. Note that the amount that would be reported as net patient service revenue is $4,500,000 (i.e., $5,000,000 - $500,000).
Selected information from the accounts of Row Co. at December 31 follows:
Total income since incorporation $420,000
Total cash dividends paid 130,000
Total value of property dividends distributed 30,000
Excess of proceeds over cost of treasury stock sold, accounted for using the cost method 110,000
In its December 31 financial statements, what amount should Row report as retained earnings?
A. $ 260,000 B. $ 290,000 C. $ 370,000 D. $ 400,000
The correct answer is: A
Explanation:
The excess of proceeds over cost of treasury stock sold, accounted for using the cost method, is credited to an appropriately titled Paid-In Capital account , such as Additional Paid-In Capital From Treasury Stock Transactions .
Income since incorporation $420,000
Less: Cash dividends (130,000)
Less: Property dividends ( 30,000)
Retained earnings, 12/31 $260,000
Retained Earnings Formula
Beg RE + Net Income- Cash Dividends- Stock Dividends = Retained Earnings
Under IFRS, what are the required classifications of the Statement of Financial Position?
A Current and non-current B Current and long-term C Cash, receivables, plant and equipment, other assets, payables, debt and other liabilities D Sales, cost of goods sold, gross profit, net income
Answer is: A
Explanation:
The classified Statement of Financial Position only requires the classifications of current and non-current. Asset and liabilities are both divided into these two classifications.
Options (B), (C) and (D) are incorrect as per above explanation.
Earnings per share data should be reported on the income statement for:
Discontinued Operations Income from continuing operations Net income A Yes Yes Yes B Yes Yes No C No No Yes D No Yes Yes
Explanation:
The correct answer is Option (A).
Earnings per share (EPS) refers to the amount of earnings attributable to each share of common stock. Publicly-held companies required to present basic EPS and dilutive EPS on:
Face of the statement of income for:
Income from continuing operations
Net income.
Either the face of statement of income or in notes to the financial statements for:
Discontinued operations.
Options (B), (C) and (D) are incorrect based on the above explanation.
Jones Corp.'s capital structure was as follows: December 31 Year 7 Year 8 Outstanding shares of stock: 110,000 110,000 Convertible 10,000 10,000 8% convertible bonds $1,000,000 $1,000,000During year 8, Jones paid dividends of $3.00 per share on its preferred stock. The preferred shares are convertible into 20,000 shares of common stock. The 8% bonds are convertible into 30,000 shares of common stock. Net income for year 8 is $850,000. Assume that the income tax rate is 30%. The diluted earnings per share for year 8 is A $5.48 B $5.66 C $5.81 D $6.26
Answer is: B
Explanation:
Earnings per share (EPS) is computed by dividing net income less preferred stock dividends by the weighted average shares of common stock outstanding. Diluted EPS adjusts this calculation to reflect all potentially dilutive securities. To compute diluted EPS for this question, the convertible securities are assumed to have been converted at the beginning of the year. The preferred stock dividend of $30,000 (10,000 × $3) is added back to the numerator (canceling out its original subtraction) and the 20,000 shares of converted common stock are added to the denominator. The bond interest expense, net of tax, of $56,000 [($1,000,000 × 8%) × (1 - 30%)] is added back to the numerator and the 30,000 shares of converted common stock are added to the denominator.
Diluted EPS = $850,000 Net income + $56,000 Bond interest (net of tax) = $5.66
110,000 (CS) + 20,000 (conv. PS) + 30,000 (conv. bonds)
A not-for-profit organization receives $150 from a donor. The donor receives two tickets to a theater show and an acknowledgment in the theater program. The tickets have a fair market value of $100. What amount is recorded as contribution revenue? A $0 B $ 50 C $100 D $150
Answer is B
Explanation:
Only that portion exceeding the fair market value of a benefit to the contributor is included in contribution revenue. ($150 - $50 = $100).
The following is Gold Corp.’s June 30, current year trial balance:
Cash overdraft $ 10,000
Accounts receivable, net $ 35,000
Inventory 58,000
Prepaid expenses 12,000
Land held for resale 100,000
Property, plant, and equipment, net 95,000
Accounts payable and accrued expenses 32,000
Common stock 25,000
Additional paid-in capital 150,000
Retained earnings 83,000
$300,000 $300,000Additional information:
Checks amounting to $30,000 were written to vendors and recorded on June 29 resulting in a cash overdraft of $10,000. The checks were mailed on July 9.
Land held for resale was sold for cash on July 15.
Gold issued its financial statements on July 31.
In its June 30 current year balance sheet, what amount should Gold report as current assets?
A
$225,000
B
$205,000
C
$195,000
D
$125,000
Answer is A
Explanation:
The cash account at 6/30 is increased by $20,000; the $30,000 of checks, less the $10,000 overdraft, written to vendors and recorded on 06/29 but not mailed until after the balance sheet date on 07/09. The land held for resale is reported as a current asset at 06/30 since it was sold for cash prior to the date the financial statements were issued.
Cash 20,000 Accounts receivable, net 35,000 Inventory 58,000 Prepaid expenses 12,000 Land held for resale 100,000 Total current assets $225,000
Valley’s community hospital normally includes proceeds from the sale of cafeteria meals in
A Deductions from dietary service expenses B Ancillary service revenues C Patient service revenues D Other revenues
Answer is D
Explanation:
Other Revenue of a healthcare entity is the usual day-to-day revenue not derived from patient care and services. It includes (1) proceeds from cafeteria meals sold, (2) revenue from educational programs, and (3) revenue from miscellaneous sources, such as from gift shops and parking lots. The proceeds from the sale of cafeteria meals do not offset dietary service expenses. Patient service revenues consist of revenue from routine services (e.g., room, board, and general nursing), other nursing services (e.g., operating, recovery, and delivery rooms), and professional services (e.g., physicians’ care, lab work, and pharmacy).
Penn Corp. paid $300,000 for the outstanding common stock of Star Co. At that time, Star had the following condensed balance sheet:
Carrying amounts
Current assets $ 40,000
Plant and equipment, net 380,000
Liabilities 200,000
Stockholders’ equity 220,000
The fair value of the plant and equipment was $60,000 more than its recorded carrying amount. The fair values and carrying amounts were equal for all other
assets and liabilities. What amount of goodwill, related to Star’s acquisition, should Penn report in its consolidated balance sheet?
A $20,000 B $40,000 C $60,000 D $80,000
Answer is A
Explanation:
Goodwill is the excess of the investment cost, plus the fair value of any noncontrolling interest, over the fair value of the identifiable net assets (INA)
acquired.
Purchase price 300,000
Current assets $ 40,000
Plant and equipment, net ($380,000 + $60,000) 440,000
Fair value of identifiable assets $480,000
Less: Liabilities assumed 200,000
Fair value of the INA acquired 280,000
Goodwill $ 20,000
In its fiscal year ended June 30, year 1, Barr College, a large private institution, received $100,000 designated by the donor for scholarships for superior students. On July 26, year 1, Barr selected the students and awarded the scholarships. How should the July 26 transaction be reported in Barr’s statement of activities for the year ended June 30, year 2?
A
As both an increase and a decrease of $100,000 in unrestricted net assets
B
As a decrease only in unrestricted net assets
C
By footnote disclosure only
D
Not reported
Answer is A
Explanation:
When the terms of a gift are met, the assets and net assets are reclassified, increasing unrestricted net assets. With the concurrent use of the assets, unrestricted net assets decreases.
Which of the following comprise functional expense categories for a nongovernmental not-for-profit organization?
A
Program services, management and general, and fund-raising
B
Membership dues, fund-raising, and management and general
C
Grant expenses, program services, and membership development
D
Membership development, professional fees, and program services
Answer is A
xplanation:
Functional Classification of Expenses: Program Services (further the mission of the organization - emergency relief, research, community services, public health, and education). Supporting Services (secondary to the mission) Fundraising expenses and G&A expenses.
G&A - Marketing, tax preparation, printing annual report, business management, budgeting.
Fund Raising - Print & mail pledge cards, maintaining donor list, merchandise sent to potential contributors, salaries of fundraisers.
Option (b) is incorrect because membership dues are part of revenues, not functional expenses. Option (c) and (d) are incorrect as per the above explanation.
A labor union, had the following expenses: Labor negotiations $500,000 Fund-raising 100,000 Membership development 50,000 Administrative and general 200,000In the statement of activity, what amount should be reported under the classification of program services? A $850,000 B $600,000 C $550,000 D $500,000
D
Explanation:
Expenses of nonprofit organizations are reported in two categories: (1) program services and (2) support services. Program services are related directly to the primary missions of the nonprofit organization. A labor union would report the cost of labor negotiations under the classification of program services. Support services do not relate to the primary missions of the nonprofit organization and include such costs as management and general administration, membership development, and fund-raising.
A not-for-profit voluntary health and welfare organization should report a contribution for the construction of a new building as cash flows from which of the following in the statement of cash flows? A Operating activities B Financing activities C Capital financing activities D Investing activities
Answer is B
Explanation:
Not-for-profit organizations follow the FASB format of statement of cash flows which does not include a capital financing activities section. For not-for-profit organizations, the description of cash flows from financing activities is expanded to include certain donor-restricted cash that must be used for long-term purposes. Contributions for construction of a capital asset would not be included in the operating or investing activities section.
In the previous year, Citizens' Health, a voluntary health and welfare organization, received a bequest of a $200,000 certificate of deposit maturing in the current year. The testator's only stipulations were that this certificate be held until maturity and that the interest revenue be used to finance salaries for a preschool program. Interest revenue for the current year was $16,000. When the certificate matured and was redeemed, the board of trustees adopted a formal resolution designating $40,000 of the proceeds for the future purchase of equipment for the preschool program. What amount should Citizen report in its current year-end current funds balance sheet as net assets designated for the preschool program? A $0 B $16,000 C $40,000 D $56,000
Answer is C
Explanation:
The voluntary health and welfare organization (VHWO) used an endowment fund to account for the certificate of deposit because the testator stipulated that the certificate be held until maturity and that the interest revenue be restricted for a special purpose (i.e., to be used to finance salaries for a preschool program). In 2000, the restrictions on the endowment fund principal lapsed (i.e., the certificate of deposit matured). Since the testator did not place any restrictions on the principal of the certificate, that amount was transferred to the current unrestricted fund. The board of trustees then adopted a formal resolution designating $40,000 of the proceeds of the certificate of deposit for the future purchase of equipment for the preschool program. Therefore, $40,000 should be reported in the current funds balance sheet as net assets designated by the governing board for the preschool program. The interest revenue is restricted per testator specifications for a special purpose. Therefore, it cannot be reported in the current funds balance sheet as net assets designated by the governing board for the preschool progra
A college’s plant funds group includes which of the following subgroups?
Renewals and replacement funds. Retirement of indebtedness funds. Restricted current funds. A I and II. B I and III. C II and III. D I only.
Answer is A
Explanation:
The plant funds group for colleges and universities includes four subgroups: (1) unexpended plant funds, (2) funds for renewals and replacements, (3) funds for retirement of indebtedness, and (4) investment in plant. Thus, a college’s plant funds group includes subgroups for (I) renewals and replacement funds and (II) retirement of indebtedness funds but not (III) restricted current funds.
Pica, a nongovernmental not-for-profit organization, received unconditional promises of $100,000 expected to be collected within one year. Pica received $10,000 prior to year end. Pica anticipates collecting 90% of the contributions and has a June 30 fiscal year end. What amount should Pica record as contribution revenue as of June 30?
A $10,000 B $80,000 C $90,000 D $100,000
Answer is C
Explanation:
An unconditional promise to give by the donor is recorded as revenue at fair value when the pledge is made after provision for allowance for the uncollectible amount. Pica anticipates collecting 90% of the contribution, 10% is an allowance for uncollectible and the rest 90% of $100,000 = $90,000 should be recorded as contribution revenue as of June 30. Option (a) and (d) are incorrect because contribution revenue should be accounted for on an accrual basis less allowance for uncollectible. Option (b) is incorrect because the receipt of $10,000 is also deducted
Kind Nurses Assoc. is a voluntary health and welfare organization. Nurses are paid to visit the homes of elderly people and are reimbursed for mileage and supplies. Which of the following items should Kind record as a support activity expense in its statement of functional expense?
A Nurses' mileage expense B Payment for nurses' employee benefits C Payment for nurses' supplies D Fund-raising costs
Explanation:
The correct answer is (D).
Fundraising expenses would be appropriately classified as a support activity expense in its statement of functional expense. Supporting activities generally include fundraising costs, administrative services, and membership development expenses. The statement of functional expenses, which is prepared by voluntary health and welfare organizations, is a two-dimensional report which reports only expenses classifying them into functional categories of program expenses and support service expenses. Program expenses are services that relate directly to the organization’s mission. Support service expenses are expenses related to management and general activities of the organization, its fundraising activities and membership development activities. The fund-raising costs of Kind Nurses Assoc. are recorded as support activity expenses in the statement of functional expense.
(A) is incorrect because a nurse’s mileage expenses are not support activities. These are program expenses. Program expenses are directly related to an organization’s mission. Kind Nurses Assoc. is paying its nurses to visit the homes of elderly people and pays reimbursement for mileage expenses making it program expenses
(B) is incorrect because Payment for nurses’employee benefits is not a support activity. It is a program expense. Payment for a nurse’s employee benefits is part of the normal compensation and benefits process.
(C) is incorrect because payments for a nurse’s supplies are also part of program services.
Funds received by a college from donors who have stipulated that the principal is nonexpendable but that the income generated may be expended by current operating funds would be a A Endowment. B Term endowment. C Not an endowment. D Quasi-endowment.
Answer is A
Explanation:
An Endowment is funds donated for which a donor or external agency has specified that the principal remain intact in perpetuity (i.e., the principal is nonexpendable).
On January 1, Read, a nongovernmental not-for-profit organization, received $20,000 and an unconditional pledge of $20,000 for each of the next four calendar years to be paid on the first day of each year. The present value of an ordinary annuity for four years at a constant interest rate of 8% is 3.312. What amount of net assets with donor restrictions is reported in the year the pledge was received?
A $ 66,240 B $ 80,000 C $ 86,240 D $100,000
Answer is A
Explanation:
Multi-year pledges: Net assets with donor restrictions to be reported at present value. $20,000 received in the current year is revenue recognized without donor restrictions. Pledges to be received in future is revenue with donor restrictions ($20,000 x 3.312 = $66,240). Option (b) is incorrect because $80,000 does not take the future pledge receivable at present value. Option (c) is incorrect because $86,240 includes current year pledge without donor restrictions received of $20,000. Option (d) is incorrect because the total pledge to be received over the multiple period from the donor, and includes pledge without donor restrictions and pledge with donor restrictions without consideration for present value of the dollar.
A nongovernmental, not-for-profit organization provided the following data in regard to $500,000 of donations received during the year:
Purchase of investments to be held in perpetuity at the donor’s request $100,000
Future repairs to the organization’s building and equipment at the donor’s request 250,000
General operations at the discretion of the board of directors 100,000
Specific program services as indicated by the donor 50,000
In order to properly reflect receipt of the donations, net assets should increase in the amount of
A
$100,000 Net Assets without Donor Restrictions and $100,000 Net Assets with Donor Restrictions
B
$150,000 Net Assets without Donor Restrictions, $250,000 Net Assets with Donor Restrictions, and $100,000 Liability
C
$100,000 Net Assets without Donor Restrictions, $400,000 Net Assets with Donor Restrictions
D
$500,000 Net Assets without Donor Restrictions
Explanation:
The correct answer is (C).
Net Assets without Donor Restrictions represent resources whose use is not restricted by donors.
The $100,000 donation for general operations to be used at the discretion of the board of directors would increase Net Assets without Donor Restrictions.
Contributions with donor-imposed restrictions are reported as Net Assets with Donor Restrictions.
Restricted support increases Net Assets with Donor Restrictions net assets.
The $250,000 for future repairs to the building and equipment, along with the $50,000 for special program services and the $100,000 donation to purchase investments to be held in perpetuity would increase Net Assets with Donor Restrictions net assets $300,000.
On December 31 of the previous year, Dahlia, a nongovernmental not-for-profit organization, purchased a vehicle with $15,000 unrestricted cash and received a donated second vehicle having a fair value of $12,000. Dahlia expects each vehicle to provide it with equal service value over each of the next five years and then to have no residual value. Dahlia has an accounting policy implying a time restriction on gifts of long-lived assets. In Dahlia's current year statement of activities, what depreciation expense should be included under changes in unrestricted net assets? A $0 B $2,400 C $3,000 D $5,400
Answer is D
Explanation:
($15,000 + $12,000 - $0) / 5 = $5,400.
The following expenditures were among those incurred by Hope University during the year:
Administrative data processing $100,000
Scholarships and fellowships 200,000
Operation and maintenance of physical plant 400,000
The amount to be included in the functional classification “Institutional Support” expenditures account is
A $100,000 B $300,000 C $500,000 D $700,000
Answer is A
Explanation:
There are separate functional classifications for expenditures “Institutional Support,” “Scholarships and Fellowships,” and “Operation and Maintenance of
Plant.” Therefore, the only expenditure in question that should be included in the functional classification “Institutional Support” is the one for administrative
data processing.
A nongovernmental not-for-profit college has a portfolio of bond investments that had an original cost of $2,000,000. The college’s board of trustees voted to hold the principal of this fund intact in perpetuity and designated the earnings to reimburse faculty for travel to academic conferences. During the year, interest of $50,000 was earned in cash. The fair value of the bonds was $1,980,000. What amount should the college report as restricted net assets at year end?
A $0 B $1,980,000 C $2,000,000 D $2,030,000
Answer is A
Explanation:
Restricted net assets represent resources that must be maintained from unrestricted net assets. Restrictions may be imposed only by a donor. The governing board of an entity may earmark assets for specific purposes as long as these do not conflict with donor conditions. These assets may be designated board-restricted in the financial statements, but they remain in the unrestricted net asset category because the board of trustees could vote to do something else with the funds.
What is the appropriate characterization of the net assets of a non-governmental not-for-profit organization?
A Residual interest B Ownership interest C Donor's interest D Equity interest
Explanation:
The correct answer is (A).
The appropriate characterization of net assets of a non-governmental not-for-profit organization is residual interest. Net assets of a non-governmental not-for-profit are defined to be the residual interest in the assets of the entity that remains after reducing its liabilities.
(B) is incorrect because ownership interest means being entitled to the ownership and profits of the entity. NPOs do not operate for profits and net assets cannot be called ownership interest.
(C) is incorrect because the donor makes a contribution by transferring assets to an NPO and the asset is used as per the instructions of the donor. The donor does not have any interest remaining in the asset unless the restrictions as set by the donor are not met.
(D) is incorrect because as there are no owners in NPOs there is no equity ownership in NPOs
RST Charities received equities securities valued at $100,000 as an gift without donor restrictions . During the year, RST received $5,000 in dividends from these securities; at year end, the securities had a fair market value of $110,000. By what amount did these transactions increase RST’s net assets?
A $100,000 B $105,000 C $110,000 D $115,000
Answer is D
Explanation:
All applicable investments are required to be measured at fair value. Gains and losses on the investments are included in the statement of activities as increases and decreases, respectively, in net assets without donor restrictions unless the use of the securities is restricted by donors. Any dividends, interest, or other investment income are to be included in the statement of activities as earned. Such amounts would be reported as adjustments to Net assets without donor restriction sunless some donor restriction exists. The increase in net assets would be $115,000; the $100,000 equity securities received as an unrestricted gift, $10,000 gain on investment, and $5,000 in dividends received on these securities.Option (a) is incorrect because $100,000 does not include increases in fair value of the securities and dividend income on the securities. Option (b) is incorrect because $105,000 ignores increases in the fair value of the securities. Option (c) is incorrect because $110,000 does not include dividend income of $5,000.
An organization of high school seniors performs services for patients at Leer Hospital. These students are volunteers and perform services that the hospital would not otherwise provide, such as wheeling patients in the park and reading to patients. Leer spent $1,500 on a recognition banquet and awards for these volunteers. Leer has no employer-employee relationship with these volunteers, who donated 4,000 hours of service to Leer during the year. At the minimum wage rate, these services would amount to $20,600, while it is estimated that the fair value of these services was $25,000. In Leer's statement of activities, what amount should be reported for donated services? A $25,000 B $20,600 C $1,500 D $0
Answer is D
Explanation:
As the volunteers perform services that Leer would not otherwise provide, Leer should not report the estimated value of donated services in the financial statements.
Health care entities should report the estimated value of donated services in the statement of revenues and expenses of general funds if the services are significant (i.e., the services would be performed by salaried personnel if donated services were not available) and measurable and the entity controls the employment and duties of the service donors (i.e., there is the equivalent of an employer-employee relationship). The question states that the volunteers perform services that Leer would not otherwise provide and that Leer has no employer-employee relationship with these volunteers. Note, however, that if these relationships did exist, the services would be recorded at their fair value of $25,000 as both expense and contribution.
A company reports the following information for year 1:
Sale of equipment
$20,000
Issuance of the company’s bonds
$10,000
Dividends paid
$5,000
Purchase of stock of another company
$2,000
Purchase of U.S. Treasury note
$2,000
Income taxes paid
$2,000
Interest income received
$500
What is the company’s net cash flow from financing activities?
A ($9,000) B $5,000 C $5,500 D $15,000
Explanation:
The correct answer is (B).
Net flows from financing activities include an inflow of $10,000 from the issuance of the company’s bonds and an outflow of $5,000 from dividends paid. Thus, there is a net inflow of $5,000 from financing activity. Sale of equipment and purchase of stock of another company are cash flows from investing activities. A U.S. Treasury note is a cash equivalent. In the statement of cash flows, the change in cash and cash equivalent is measured. Thus, even though a purchase of a 3 month U. S. Treasury bill decreases cash, cash equivalents increase. There is no net change in the amount of cash and cash equivalents hence the transaction is not reported on the statement of cash flows. Income taxes paid and interest income received are cash flows from operating activities
A voluntary health and welfare organization received a cash donation in year 1 from a donor specifying that the amount donated be used in year 3. The cash donation should be accounted for as
A
Revenue in year 1.
B
Revenue in year 1, year 2, and year 3, and as a deferred credit in the balance sheet at the end of year 2 and year 1.
C
Revenue in year 3, and no deferred credit in the balance sheet at the end of year 1 and year 2.
D
Revenue in year 3, and as a deferred credit in the balance sheet at the end of year 1 and yea
Answer is A
Explanation:
Contributions received in advance of the year the donor intends them to be used—even if usable then for unrestricted purposes—are initially recorded in a “restricted support” account. They should be accounted for as support in the year the unconditional promise to give is made. The term “deferred” is no longer applied to donations in non-governmental nonprofit accounting.
The Cats and Dogs League was organized as a non-governmental, not-for-profit organization. The League received a pledge of $10,000 to be used to build an addition to the kennel. This donation will not be received for three years. How should this pledge be recorded?
A
It should not be accounted for until it is received
B
As restricted support of $10,000
C
As a conditional promise to give of $10,000
D
As restricted support of the present value of $10,000
Explanation:
The correct answer is (D).
Pledges to be received at a future date are recognized as Net Assets with Donor Restrictions i.e. restricted support as there is a time restriction with a corresponding debit to accounts receivable.
If collection is doubtful, an allowance can be recognized for the same.
Pledges to be used to build an addition to the kennel that would be received in 3 years would be recorded as restricted support with donor restrictions at its present value.
JE:
Pledge Receivable XXX [PV of $10,000]
Net Assets with Donor Restrictions XXX [PV of $10,000]
Cyan Corp. issued 20,000 shares of $5 par common stock at $10 per share. On December 31 of the previous year, Cyan’s retained earnings were $300,000. In
March of the current year, Cyan reacquired 5,000 shares of its common stock at $20 per share. In June of the current year, Cyan sold 1,000 of these shares to
its corporate officers for $25 per share. Cyan uses the cost method to record treasury stock. Net income for the current year ended December 31 was $60,000.
At December 31 of the current year, what amount should Cyan report as retained earnings?
A $360,000 B $365,000 C $375,000 D $380,000
Explanation:
Under the cost method, treasury stock is recorded and carried at the acquisition cost of $20 per share. Cyan sold some of the shares for $5 per share more than
the acquisition cost. This excess is credited to a Paid-In-Capital From Treasury Stock account and Retained Earnings is not affected. If Cyan sold any of the
shares for less than the acquisition cost, the deficit would first be charged to any existing balance in the Paid-In-Capital From Treasury Stock account, and the
excess, if any, would be charged against Retained Earnings .
Retained Earnings, Dec. 31, previous year $300,000
Net Income 60,000
Retained Earnings, Dec. 31, current year
For a company to obtain a retail business license in a particular state, the company is required to pay the state the equivalent of three months of sales taxes on its projected retail sales. This amount is fully refundable after five years, provided the company has filed all required sales tax returns and paid all sales taxes due. Initially the company should report the payment related to this licensing requirement as A An expense B A current asset C A noncurrent liability D A noncurrent asset
Explanation:
Initially, the company should report the payment related to the licensing requirement as a noncurrent asset. The company will receive the monies back, provided it meets the requirements, so it is considered an asset. Because the refund period is in 5 years, not within one year, it is considered a noncurrent asset.
The purpose of the IASB Conceptual Framework includes doing which of the following?
Assist the Board in the development of future IFRS
Assist preparers of financial statements in applying IFRS
Assist auditors in forming an opinion on whether financial statements comply with IFRS
A
I only
B
I and II only
C
I and III only
D
I, II, and III
Explanation:
The purpose of the IASB Conceptual Framework includes assisting, among others, the Board in the development of future IFRS, preparers of financial statements in applying IFRS, and auditors in forming an opinion on whether financial statements comply with IFRS.
How should a nongovernmental not-for-profit organization report investments in its financial statements?
A
Historical cost with no gains or losses reported.
B
Par value with gains and losses reported in the statement of activities.
C
Fair value with gains and losses reported in the statement of activities.
D
Amortized value with gains and losses reported in the statement of comprehensive income.
Explanation:
A nongovernmental not-for-profit organization should measure and report its investments in the financial statements at fair value. Gains and losses on the investments are included in the statement of activities as increases and decreases, respectively, in net assets without donor restrictions unless explicitly restricted by the donor or law. Any dividends, interest, or other investment income are to be included in the statement of activities as earned.Option (a), (b) and (d) are incorrect as per the above explanation.
A company has a total revenue of $1,000,000, profits of $90,000, total assets of $10,000,000, and total liabilities of $5,000,000. Operating segment Sun has revenues of $95,000, profit of $10,000, assets of $900,000, and liabilities of $550,000. Which of the following tests makes Sun a reportable segment?
A Revenue test B Profit test C Asset test D Liability test
Explanation:
The correct answer is (B).
Tests for reportable segments:
10% Threshold Tests - A segment is reportable if it contributes at least 10% of the overall total of one or more of the following - (1) Revenue, (2) Profit or loss, (3) Assets.
75% test for Consolidated Revenues only.
Revenues of at least 10% of total revenues for all reported operating segments = $95,000 / $1,000,000 = 9.5%
Profit or loss of at least 10% of the combined profit or loss for all operating segments = $10,000 / $90,000 = 11%
Assets of at least 10% of the combined revenues, profit or loss, or assets for all operating segments = $10,000 / $10,000,000 = 9%
The profit test shows that Sun’s profits are > 10% of total profits. Sun is a reportable segment.
A labor union, had the following receipts:
Per capita dues $680,000 Initiation fees 90,000 Sales of organizational supplies 60,000 Nonexpandable gift restricted by management for loan purposes for 10 years 30,000 Nonexpandable gift restricted by donor for loan purposes in perpetuity 25,000 In the statement of activity, what amount should be reported as restricted revenue?
A $123,000 B $93,000 C $55,000 D $25,000
Explanation:
For financial statement purposes, only donors can restrict revenues or net assets.
Management or the Board of Directors may earmark funds but are free to change their designations at any time.
A segment of Ace, Inc., was discontinued during the current year. In comparative financial statements for the previous year, Ace’s loss on disposal should
A
Exclude comparative financial statements for contingent product warranty obligation costs.
B
Include operating losses during the current fiscal year.
C
Exclude additional pension costs associated with the decision to dispose.
D
Include operating losses of the previous fiscal year up to the date a disposal plan was adopted.
D or B Maybe
Explanation:
In the period in which a component of an entity either has been disposed of or is classified as held for sale, the income statement of a business enterprise for current and prior periods shall report the results of operations of the component in discontinued operations in the period(s) in which they occur. Amounts reported in discontinued operations, include, the resolution of contingencies that arise from product warranty obligations and the settlement of employee benefit plan obligations (pension and other post-employment benefits), provided tha
Unless a majority of the segment's \_\_\_\_\_\_\_\_\_\_\_\_\_\_\_\_ are from \_\_\_\_\_\_\_\_\_\_\_\_\_, an enterprise must report interest revenue separately from interest expense for each reportable segment. A Revenues; operating income B Expenses; operating income C Revenues; interest D None of the above
C
Explanation:
An enterprise must report interest revenue separately from interest expense for each reportable segment unless a majority of the segment’s revenues are from interest.
Dunn, Inc. had 200,000 shares of $20 par common stock and 20,000 shares of $100 par, 6%, cumulative, convertible preferred stock outstanding for the entire current year ended December 31. Each preferred share is convertible into five shares of common stock. Dunn's net income for the year was $840,000. For the current year ended December 31, the diluted earnings per share is A $2.40 B $2.80 C $3.60 D $4.20
B
Explanation:
Earnings per share (EPS) is computed by dividing net income less preferred stock dividends by the weighted average shares of common stock outstanding. Diluted EPS adjusts this calculation to reflect all potentially dilutive securities. Since the computation of diluted earnings per share assumes all potentially dilutive convertible securities have been converted, Dunn’s diluted earnings per share should be calculated based on the net income figure without deducting preferred dividends. The convertible preferred stock is assumed converted at the beginning of the year. The preferred stock dividend of $120,000 (20,000 × $100 × 6%) is added back to the numerator (canceling out its original subtraction) and the 100,000 (20,000 × 5) shares of converted common stock are added to the denominator.
$840,000 Net Income = $2.80
200,000 Common + 100,000 Convertible preferred
A company has the following items on its year-end trial balance:
Net sales $500,000 Common stock 100,000 Insurance expense 75,000 Wages 50,000 Cost of goods sold 100,000 Cash 40,000 Accounts payable 25,000 Interest payable 20,000 What is the company's gross profit?
A $230,000 B $275,000 C $400,000 D $500,000
Explanation:
Gross Profit = Net sales - COGS ($500,000 - $100,000) = $400,000.
Option (A) is incorrect because insurance expense, wages, accounts payable and interest payable have been included.
Option (B) is incorrect because insurance expense, wages have been included.
Option (D) is incorrect because it does not subtract the Cost of Goods Sold (COGS).
Chape Co. had the following information related to common and preferred shares during the year: Common shares outstanding 1/1 700,000 Common shares repurchased 3/31 20,000 Conversion of preferred shares 6/30 40,000 Common shares repurchased 12/1 36,000Chape reported net income of $2,000,000 at December 31. What amount of shares should Chape use as the denominator in the computation of basic earnings per share? A 684,000 B 700,000 C 702,000 D 740,000
C
Explanation:
Basic earnings per share (EPS) is computed by dividing income available to common stockholders by the weighted-average number of shares outstanding during the period. Shares issued during the period and shares reacquired during the period are weighted for the portion of the period they were outstanding. Stock dividends, stock splits, and reverse stock splits change the total number of shares outstanding but not the proportionate shares outstanding. For this reason, stock dividends, stock splits, and reverse stock splits are reflected retroactively for all periods presented. The weighted-average number of shares used to calculate basic EPS is 702,000. It’s 100% of the 644,000 shares that have been outstanding all year (700,000 - 20,000 - 36,000), plus 3/12 of the 20,000 shares repurchased on March 31 (5,000), plus 11/12 of the shares repurchased on December 1 (33,000), and 50% of the 40,000 preferred shares converted at June 30 (20,000).
During the current year, Onal Co. purchased 10,000 shares of its own stock at $7 per share. The stock was originally issued at $6. The firm sold 5,000 of the
treasury shares for $10 per share. The firm uses the cost method to account for treasury stock. What amount should Onal report in its income statement for
these transactions?
A $0. B $ 5,000 gain. C $10,000 loss. D $15,000 gain.
A
Explanation:
Treasury stock is the corporation’s common or preferred stock that has been reacquired by purchase, by settlement of an obligation to the corporation, or
through donation. The cost method views the purchase and subsequent disposition of stock as one transaction. The treasury stock is recorded (debited),
carried, and reissued at the acquisition cost. If the stock is reissued at a price in excess of the acquisition cost, such as in this situation, the excess is credited to
an appropriately titled paid-in capital account. There are no gains or losses reported in the income statement for these transactions.
What is the purpose of reporting comprehensive income?
A
To summarize all changes in equity from nonowner sources.
B
To reconcile the difference between net income and cash flows provided from operating activities.
C
To provide a consolidation of the income of the firm’s segments.
D
To provide information for each segment of the business.
Explanation:
Comprehensive income includes all the changes in equity during a period except those resulting from investments by owners & distribution to owners.
Option (B) is incorrect because it pertains to statement of cash flows.
Option (C) is incorrect because it pertains to segment reporting.
Option (D) is incorrect because it also pertains to segment reporting.
Which of the following should be disclosed in a summary of significant accounting policies?
A
Basis of consolidation.
B
Concentration of credit risk of financial instruments.
C
Composition of plant assets.
D
Adequacy of pension plan assets in relation to vested benefits.
A
Explanation:
The disclosure of accounting policies is considered an integral part of the financial statements. No specific disclosure format is required. A separate note, or a summary preceding the notes entitled Summary of Significant Accounting Policies is preferred. The accounting policy disclosures should identify and describe the principles and methods that materially affect the financial position and operations. Disclosure should include policies involving a choice of alternative acceptable policies, policies peculiar to the industry, and unusual applications of acceptable principles. Disclosure requirements include: criteria determining which investments are treated as cash equivalent; basis of accounting for loans and trade receivables; method of recognizing interest income on loan and trade receivables; depreciation methods; inventory pricing methods; methods of recognizing profit on long-term construction contracts; and basis of consolidation.
Which of the following characteristics of accounting information primarily allows users of financial statements to generate predictions about an organization? A Reliability B Timeliness C Neutrality D Relevance
D
Explanation:
Fundamental qualitative characteristics of information are relevance and faithful representation. Information is relevant if it is capable of making a difference in a decision by helping users to form predictions about the outcomes of past, present, and future events or to confirm or correct prior expectations. Components of relevance are predictive value, confirmatory value, or both. Reliability is not a characteristic of useful financial information. Timeliness is a characteristic that enhances the usefulness of information that is relevant and faithfully represented. Neutrality is a characteristic of faithful representation.
Port Inc. owns 100% of Salem, Inc. On January 1 of the current year, Port sold Salem delivery equipment at a gain. Port had owned the equipment for two
years and used a five-year straight-line depreciation rate with no residual value. Salem is using a three-year straight-line depreciation rate with no residual
value for the equipment. In the consolidated income statement, Salem’s recorded depreciation expense on the equipment for the year will be decreased by
A 20% of the gain on sale. B 33 1/3% of the gain on sale. C 50% of the gain on sale. D 100% of the gain on sale.
B
Explanation:
In the consolidated balance sheet, the machine’s cost and accumulated depreciation must be based upon the cost of the machine to the consolidated entity.
Likewise, consolidated depreciation expense must be reduced by the excess depreciation recorded by the purchasing affiliate. Since the machine’s useful life
on the date of sale is 3 years, Salem’s depreciation expense must be decreased by 1/3, or 33-1/3%, of the gain for consolidated purposes.
The ____________ method of accounting for treasury stock transactions views the purchase and subsequent disposition of stock as one transaction.
A Cost B Par value C Acquisition D None of the above
A
Explanation:
The cost method of accounting for treasury stock views the purchase and subsequent disposition of stock as one transaction.
On July 1, year 2, a company decided to adopt IFRS. The company’s first IFRS reporting period is as of and for the year ended December 31, year 2. The company will present one year of comparative information. What is the company’s date of transition to IFRS?
A January 1, year 1 B January 1, year 2 C July 1, year 2 D December 31, year 2
A
Explanation:
The starting point in IFRS 1 is an opening IFRS statement of financial position prepared at the date of transition to IFRS. The date of transition to IFRS is defined as the beginning of the earliest period for which an entity presents full comparative information under IFRS in its first IFRS financial statements. For entities that present one year of comparative information in their financial reports, the date of transition is the first day of the comparative period. In the given situation the company did present one year of comparative information so the transition date to IFRS would be January1, year 1, the first day of the comparative period.
A voluntary health and welfare organization received a $700,000 permanent endowment during the year. The donor stipulated that the income and investment appreciation be used to maintain its senior center. The endowment fund reported a net investment appreciation of $80,000 and investment income of $50,000. The organization spent $60,000 to maintain its senior center during the year. What amount of change in net assets with donor restrictions should the organization report?
A $ 50,000 B $ 70,000 C $130,000 D $770,000
Explanation:
The correct answer is (D).
$700,000 endowment received by the voluntary health and welfare organization would be classified as an increase in Net Assets with Donor Restrictions. Income of $50,000 and investment appreciation of $80,000 will also increase Net Assets with Donor Restrictions. So, total increase in Net Assets with Donor Restrictions is $830,000 [$700,000 + $50,000 + 80,000]. The actual $60,000 spent represents a decrease in Net Assets with Donor Restrictions. Hence, total changes in Net Assets with donor restrictions is $770,000 [$830,000 - $60,000].
The following trial balance of Trey Co. at December 31 of the current year has been adjusted except for income tax expenses.
Dr. Cr.
Cash $ 550,000
Accounts receivable, net 1,650,000
Prepaid taxes 300,000
Accounts payable $ 120,000
Common stock 500,000
Additional paid-in capital 680,000
Retained earnings 630,000
Foreign currency translation adjustment 430,000
Revenues 3,600,000
Expenses 2,600,000
$5,530,000 $5,530,000
During the year, estimated tax payments of $300,000 were charged to prepaid taxes. Trey has not yet recorded income tax expense. There were no differences between financial statement and income tax income, and Trey’s tax rate is 30%.
Included in accounts receivable is $500,000 due from a customer. Special terms granted to this customer require payment in equal semi-annual installments of $125,000 every April 1 and October 1.
In Trey’s December 31 year-end balance sheet, what amount should be reported as total current assets?
A $1,950,000 B $2,200,000 C $2,250,000 D $2,500,000
A
$1,950,000 is correct. Current assets are assets that are collectible within one year. The sum of the stated current assets is $2,500,000 ($550,000+$1,650,000+$300,000). However, once the current tax bill is calculated, the prepaid taxes of $300,000 are transferred into a tax expense account to cover the $300,000 in current year tax expense. In addition, $250,000 of the special accounts receivable is not due for over one year and is, therefore, non-current. Therefore, current assets should be $1,950,000 ($2,500,000-$300,000-$250,000).
How should a nongovernmental not-for-profit organization classify gains and losses on investments purchased with assets with donor restrictions?
A
Gains may not be netted against losses in the statement of activities.
B
Gains and losses can only be reported net of expenses in the statement of activities.
C
Unless explicitly restricted by donor or law, gains and losses should be reported in the statement of activities as increases or decreases in net assets without donor restrictions.
D
Unless explicitly restricted by donor or law ,gains and losses should be reported in the statement of activities as increases or decreases in net assets with donor restrictions.
Explanation:
C
Explanation:
The income generated from assets with donor restrictions may be spent and should be reported in net assets without donor restrictions unless explicitly restricted by donor or law (in which case,would be reported under assets with donor restrictions). Option (a), (b) and (d) are incorrect as per the above explanation.
Belle, a nongovernmental not-for-profit organization, received funds during its annual campaign that were specifically pledged by the donor to another nongovernmental not-for-profit health organization. How should Belle record these funds?
A
Increase in assets and increase in liabilities.
B
Increase in assets and increase in revenue.
C
Increase in assets and increase in deferred revenue.
D
Decrease in assets and decrease in fund balance.
A
Explanation:
Donations received in cash will increase the assets as well as increase liabilities as they are payable to another NFP.
Option (B) is incorrect because donation increases liability as pledged to another NFP and does not increase the revenue.
Option (C) is incorrect because donation increases liability as pledged to another NFP and does not increase the deferred revenue.
Option (D) is incorrect because assets will increase and liabilities will increase; and fund balance will not decrease.
Mann, Inc. had 300,000 shares of common stock issued and outstanding at December 31, year 1. On July 1, year 2, an additional 50,000 shares of common stock were issued for cash. Mann also had unexercised stock options to purchase 40,000 shares of common stock at $15 per share outstanding at the beginning and end of year 2. The average market price of Mann’s common stock was $20 during year 2.
What is the number of shares that should be used in computing basic earnings per share for the year ended December 31, year 2?
A 325,000 B 335,000 C 360,000 D 365,000
A
Explanation:
Basic EPS is computed by dividing income available to shareholders (IAC) by the weighted average number of shares outstanding during the period. Weighted average number of shares is (300,000 × 12/12) + (50,000 × 6/12) = 325,000.
Unusual or infrequent items are reported in the
A Income Statement B Financial Statement Footnotes C Income Statement or Financial Statement Footnotes D None of the Above
C
Explanation:
For unusual or infrequent Items, there are two options for reporting:
Income Statement (above Income from Continuing Operations)
Footnotes to Financial Statements
Unusual or infrequent Items get Retrospective or Prospective treatment in the financial statements.
For a marketable debt securities portfolio classified as available-for-sale, which of the following amounts should be included in the period’s net income?
Unrealized holding losses during the period Realized gains during the period Changes in the Market Adjustment account during the period A III only. B II only. C I and II. D I, II, and III.
B
Explanation:
Unrealized holding gains and losses for AFS debt securities are excluded from earnings and reported in other comprehensive income until realized.
Any changes in the Market Adjustment account would have no effect on net income.
Which of the following characteristics means that information is reasonably free from error and bias?
A Faithful representation B Relevance C Consistency D Predictive value
Explanation:
The correct answer is (A).
Faithful representation is a primary qualitative characteristic of financial statements and consists of the following three ingredients:
Error free (no errors/omissions).
Neutrality (no bias).
Completeness (complete).
Faithful representation is the characteristic that suggests that information is reasonably free from error and bias.
Dee City’s community hospital normally includes proceeds from the sale of cafeteria meals in
A Patient service revenues. B Other revenues. C Ancillary service revenues. D Deductions from dietary service expenses.
B
Explanation:
Other revenues of a health care entity are the usual day-to-day revenues that are not derived from patient care and services, and generally, include (1) proceeds
from the sale of cafeteria meals, (2) revenue from educational programs, and (3) revenues from miscellaneous sources, such as revenue from gift shops and
parking lots.
Within the context of the qualitative characteristics of accounting information, which of the following is a fundamental qualitative characteristic?
A Relevance B Timeliness C Comparability D Feedback value
Explanation:
The correct answer is (A).
As per the US GAAP, FASB conceptual framework, the primary or fundamental qualitative characteristics of accounting and reporting are Relevance and Faithful Representation.
Relevance
Predictive value
Confirmatory value
Materiality
Faithful Representation
Freedom from error
Neutrality
Completeness
Thus, relevance is a fundamental qualitative characteristic.
The enhancing qualitative characteristics are comparability, understandability, timeliness, and verifiability.
Governmental financial reporting should provide information to assist users in which situation(s)?
Making social and political decisions. Assessing whether current-year citizens received services but shifted part of the payment burden to future-year citizens. A I only B II only C Both I and II D Neither I nor II
C
Explanation:
Governmental financial reporting is used in making social and political decisions. Because interperiod equity is important, financial reporting should help users assess whether current-year revenues are sufficient to provide current services or whether future taxpayers are assuming the burden of previously provided services.
For the year ended December 31, Ion Corp. had cash inflows of $25,000 from the purchases, sales, and maturities of held-to-maturity securities and $40,000 from the purchases, sales, and maturities of available-for-sale debt securities. What amount of net cash from investing activities should Ion report in its cash flow statement?
A $0 B $25,000 C $40,000 D $65,000
Explanation:
Investing activities include the following:Principal collections or loans made by the entity (interest and dividends received are operating). Acquisition or disposal of Available-for-Sale (AFS) debt securities or Held-to-Maturity (HTM) investments (not trading). Acquisition or disposal of PP&E and intangibles.The net cash from investing activities is reported at $65,000
Ref
Cash flows from investing activities
Change
a
Purchases, sales and maturities of HTM
$25,000
b
Purchases, sales and maturities of AFS debt securities
40,000
c
Net cash provided (used) by investing activities (a+b)
$65,000
Option (a) is incorrect because net cash inflow from investing activities is reported at $65,000.Option (b) is incorrect because it does not include cash inflow from AFS debt Option (c) is incorrect because it does not include cash inflow from HTM securities.
In financial reporting of segment data, which of the following must be considered in determining if an industry segment is a reportable segment?
Sales to
unaffiliated customers Intersegment sales
A Yes Yes
B Yes No
C No Yes
D No No
A
Explanation:
Reportable segments include operating segments that exceed the quantitative thresholds. An enterprise shall report separately information about an operating segment that meets any of the following three quantitative thresholds: (1) its reported revenue, including both sales to external customers and intersegment sales or transfers, is 10% or more of the combined revenue, internal and external, of all operating segments, (2) the absolute amount of its reported profit or loss is 10% or more of the greater, in absolute amount, of (a) the combined reported profit of all operating segments that did not report a loss or (b) the combined reported loss of all operating segments that did report a loss, or (3) its assets are 10% or more of the combined assets of all operating segments.
A not-for-profit hospital issued long-term tax-exempt bonds for the hospital’s benefit. The hospital is responsible for the liability. Which fund may the hospital use to account for this liability?
A Enterprise B Specific purpose C General D General long-term debt account group.
C- General
Explanation:
Healthcare entities have only two categories of funds: (1) the general funds and (2) the donor-restricted funds. The general fund is used to account for all assets and liabilities that are not required to be accounted for in a donor-restricted fund. Since the bonds in question were issued for the hospital’s benefit and are unrelated to any donor-restricted assets, they should be accounted for in the general fund. Both enterprise funds and the general long-term debt account group are utilized by state and local governments rather than by nonprofit healthcare entities .Specific purpose funds are a type of donor-restricted funds that are used to account for resources restricted by donors and grantors for specific operating purposes.
If no profit and loss sharing arrangement is specified in the partnership agreement, the Revised Uniform Partnership Act (RUPA) requires which of the following?
A
Profit and loss should be based on ownership interest.
B
Profit and loss should be shared equally.
C
Profit and loss should be based on the ratio of partners’ capital balances.
D
The RUPA is silent in this area, and does not specify what method should be used.
B
Explanation:
If no profit and loss sharing arrangement is specified in the partnership agreement, the RUPA requires that profits and losses be shared equally.
Which of the following is not commonly considered to be a part of the Basic Information Package (BIP) or required financial disclosures for SEC annual filings?
A
Management’s discussion and analysis for financial condition and results of operations
B
Two-year comparative financial information
C
Indication of significant adverse contingencies
D
Data explaining the circumstances applicable to a change in independent accountant during the prior two years
B
Explanation:
Financial disclosures for SEC filings are referred to as the Basic Information Package (BIP). BIP requirements are common to 10-K and to the annual report to shareholders. Two-year comparative financial information is not commonly considered to be part of the BIP. The BIP does consist of five-year comparative selected financial data, management’s discussion and analysis of the financial condition and results of operations, and data explaining the circumstances applicable to a change in the registrant’s independent accountant during the prior two years
Fenn Museum, a non-governmental not-for-profit organization, had the following balances in its statement of functional expenses:
Education
$300,000
Fundraising
$250,000
Management and General
$200,000
Research
$50,000
What amount should Fenn report as expenses for support services?
A $350,000 B $450,000 C $500,000 D $800,000
B
Explanation:
Functional Expenses for Support Services: $450,000 = ( $250,000 + $200,000);
fundraising and management & general expenses qualify as support services.
Supporting Services:- (secondary to the mission) - Fundraising expenses and G&A expenses. Fund Raising: Print & mail pledge cards, maintaining donor list, merchandise sent to potential contributors, salaries of fundraisers. Option (a) is incorrect because $350,000, is the sum of education and research expense which are functional expenses for program services. Option (c) is incorrect because education expense which is a program service is included and fundraising expenses are excluded which are expenses for support services. Option (d) is incorrect because program services functional expenses of $350,000, which is the sum of education and research is included, which should be excluded.
Which of the following is correct concerning financial statement disclosure of accounting policies?
A
Disclosures should be limited to principles and methods peculiar to the industry in which the company operates.
B
Disclosure of accounting policies is an integral part of the financial statements.
C
The format and location of accounting policy disclosures are fixed by generally accepted accounting principles.
D
Disclosures should duplicate details disclosed elsewhere in the financial statements.
B- Its an integral part of F/S
Explanation:
Disclosure of all significant accounting policies in an enterprise’s financial statements is required. Thus, disclosure of accounting policies is an integral part of the financial statements. Disclosures should not be limited to principles and methods peculiar to the industry in which it operates, but instead include policies involving a choice of alternative acceptable polices, policies peculiar to that particular industry, and unusual applications of acceptable principles. There is no specific required format and location of accounting policy disclosures. Financial statement disclosure of accounting policies should not duplicate details presented elsewhere as part of the financial statements.
Which of the following information should be disclosed as supplemental information in the statement of cash flows?
Cash flow per share Conversion of debt to equity A Yes Yes B Yes No C No Yes D No No
C
Explanation:
Financial statements shall not report an amount of cash flow per share. The conversion of debt to equity is a noncash financing activity because the transaction affects the enterprise’s liabilities but it does not result in cash receipts or payments during the period. The conversion of debt to equity should be disclosed as supplemental information in the statement of cash flows.Options (a), (b) and (d) are incorrect as per above explanation.
Which of the following accounts would appear in the plant fund of a not-for-profit private college?
Fuel inventory for power plant Equipment A Yes Yes B No Yes C No No D Yes No
B
Explanation:
The asset accounts in the Investment in Plant subgroup of the Plant Funds group of a college contain the carrying amounts of the institution’s fixed assets. Therefore, the equipment would be reported in the Investment in Plant subgroup of the Plant Funds group of the college. The fuel inventory for the college’s power plant should be reported in the Unrestricted Current Funds under Inventory of Materials and Supplies.
In a statement of activities of a voluntary health and welfare organization, contributions to the building fund should A Be included as an element of support B Be included as an element of revenue C Be included as an element of other changes in net assets D Not be included
A
Explanation:
The contributions to the building fund are included as support in the statement of activities of a voluntary health and welfare organization.
Flax Corp. uses the direct method to prepare its statement of cash flows. Flax's trial balances at December 31, year 2 and year 1, are as follows: December 31 Year 2 Year 1 Debits: Cash $35,000 $32,000 Accounts receivable 33,000 30,000 Inventory 31,000 47,000 Property, plant, and equipment 100,000 95,000 Unamortized bond discount 4,500 5,000 Cost of goods sold 250,000 380,000 Selling expenses 141,500 172,000 General and administrative expenses 137,000 151,300 Interest expense 4,300 2,600 Income tax expense 20,400 61,200 $756,700 $976,100 Year 2 Year 1 Credits: Allowance for uncollectible accounts $1,300 $1,100 Accumulated depreciation 16,500 15,000 Trade accounts payable 25,000 17,500 Income taxes payable 21,000 27,100 Deferred income taxes 5,300 4,600 8% callable bonds payable 45,000 20,000 Common stock 50,000 40,000 Additional paid-in capital 9,100 7,500 Retained earnings 44,700 64,600 Sales 538,800 778,700 $756,700 $976,100 Flax purchased $5,000 in equipment during year 2. Flax allocated one-third of its depreciation expense to selling expenses and the remainder to general and administrative expenses. What amounts should Flax report in its statement of cash flows for the year ended December 31, year 2, for cash paid for income taxes? A $25,800 B $20,400 C $19,700 D $15,000
A
Explanation:
The amount of cash paid for income taxes is determined as follows:
Income tax expense $20,400 Add: decrease in income taxes payable ($27,100 - $21,000) 6,100 Less: increase in deferred income taxes ($5,300 - $4,600) (700) Cash paid for income taxes $25,800Since income taxes payable decreased from the beginning to the end of the year, the amount of cash paid for income taxes is greater than income tax expense reported on an accrual basis by this amount. The increase in deferred income taxes is a noncash expense (i.e., it increases income tax expense but has no affect on cash), therefore, it is subtracted from income tax expense to determine the amount of cash paid for income taxes.
Which of the following statements best describes an operating procedure for issuing a new Financial Accounting Standards Board (FASB) standard?
A
The emerging issues task force must approve a discussion memorandum before it is disseminated to the public.
B
The exposure draft is modified per public opinion before issuing the discussion memorandum.
C
A new standard is issued only after a majority vote by the members of the FASB.
D
A new FASB standard can be rescinded by a majority vote of the AICPA membership.
C
Explanation:
A new Financial Accounting Standards Board (FASB) standard is issued only after a majority vote by the members of FASB. The emerging issues task force assists the FASB in improving financial reporting through the timely identification, discussion, and resolution of financial accounting issues within the framework of existing authoritative literature. It does not approve a discussion memorandum before it is disseminated to the public. A discussion memorandum is prepared before any deliberations on a new major project and thus before any exposure draft. FASB standards are not rescinded by the AICPA member’s votes.
In the statement of cash flows, advances from related parties would be reflected in which of the following sections? A Operating B Investing C Financing D Either A. or B., depending upon the amount
C
Explanation:
Advances from related parties, like other advances, should be reflected in the statements as financing activities, and repayments of the advances should also be reflected in the statements within the financing section.
Polk Co. acquires a forklift from Quest Co. for $30,000. The terms require Polk to pay $3,000 down and finance the remaining $27,000. On March 1, year 1, Polk pays the $3,000 down and accepted delivery of the forklift. Polk signed a note that requires Polk to pay principal payments of $1,000 per month for 27 months beginning July 1, year 1. What amount should Polk report as an investing activity in the statement of cash flows for the year ended December 31, year 1?
A $ 3,000 B $ 9,000 C $12,000 D $30,000
A
Explanation:
Cash flows from investing activities include 1) making and collecting loans (excluding those acquired specifically for resale), 2) acquiring and disposing of property, plant and equipment, and other productive assets, and 3) purchases, sales, and maturities of debt and equity securities (excluding those acquired specifically for resale. Borrowing money and repaying amounts borrowed, or otherwise settling the obligation represents cash flows from financing activities. Polk should report $3,000 in investing activities and $27,000 in financing activities. Options (b), (c) and (d) are incorrect as per the above explanation.
Is the recognition of depreciation expense required for public colleges and private not-for-profit colleges? Public Private A No Yes B No No C Yes Yes D Yes No
C
Explanation:
All nongovernmental nonprofit organizations must record depreciation on most long-lived tangible assets.
Which of the following is a true statement related to electronic filing of required SEC submissions?
A
Regulation S-X is the separate regulation that contains the rules prescribing electronic filing and the procedures for such filings.
B
Regulation S-T requires all SEC filings to be submitted electronically.
C
Unanticipated technical difficulties can result in a temporary hardship exemption for electronic filing.
D
Only temporary hardship exemptions are available for electronic filing requirements.
I DONT KNOW C
Explanation:
Rule 201 of Regulation S-T provides a temporary hardship exemption for electronic filers, generally for unanticipated technical difficulties in submitting an electronic document. The exemption may be appropriate, for example, for a particular document that a filer is unable to file electronically because of problems with the filer’s computer equipment that had been used previously to transmit either test or required electronic filings successfully.
Which of the following must be included in a company’s summary of significant accounting policies in the notes to the financial statements?
A Description of current year equity transactions B Summary of long-term debt outstanding C Schedule of fixed assets D Revenue recognition policies
D
Explanation:
The summary of significant accounting policies should disclose policies involving a choice of alternative acceptable polices, policies peculiar to that particular industry, and unusual applications of acceptable principles. This includes revenue recognition policies. The disclosure of accounting policies should not duplicate details presented elsewhere as part of the financial statements. Descriptions of transactions, summaries of debt, and schedules of assets are not policies and are located elsewhere in the financial statements.
Bard Co. owned several subsidiaries on December 31. The following table shows each subsidiary’s total liabilities, excluding intercompany transactions, and percentage of stock owned by Bard:
Subsidiary Total Liabilities % owned Brock Co. $4,000,000 70 Harlson Co. 2,000,000 48 Porter Co. 7,000,000 80 Nortin Co. 5,000,000 100 What amount should Bard include as liabilities in its consolidated balance sheet at December 31?
A $5,000,000 B $12,000,000 C $16,000,000 D $18,000,000
C
Explanation:
If the percentage of stock owned is greater than 50%, consolidated financial statements should be reported. Bard Co., owns Stock in Brock Co., Porter Co., and Nortin Co. greater than 50%, hence the liabilities should be consolidated and reported at $16,000,000 ($4,000,000+$7,000,000+5,000,000).
Option (A) is incorrect because $5,000,000 is only Nortin Co’. liabilities where Brad co. has 100% ownership. It should also include Brock Co. and Porter Co.’s liabilities.
Option (B) is incorrect because $12,000,000 is the liability for Nortin Co and Porter Co., Brad Co. should also include Brock Co.’s liabilities.
Option (D) is incorrect because $18,000,000 includes Harlson Co.’s liabilities which should be excluded in the consolidated balance sheet as the ownership is less than 50%.
Which of the following factors influences governmental generally accepted accounting and reporting principles?
A
The lack of SEC oversight for municipal financial instruments
B
State statutes that created the Governmental Accounting Standards Board (GASB)
C
Governmental structure and its mission to provide critical public services
D
Population levels for individual governments
C
Explanation:
U.S. state and local governments reflect the federal system of three branches of government plus the separation of powers. The essential mission of government is delivering services, financed with taxes.
Deed Co. owns 2% of Beck Cosmetic Retailers. A property dividend by Beck consisted of merchandise with a fair value lower than the listed retail price. Deed in turn gave the merchandise to its employees as a holiday bonus.
How should Deed report the receipt and distribution of the merchandise in its statement of cash flows?
A
As both an inflow and outflow for operating activities
B
As both an inflow and outflow for investing activities
C
As an inflow for investing activities and outflow for operating activities
D
As a non cash activity
D
Explanation:
These transactions represent transfer of non-cash activity. Neither the receipt of property as a dividend nor the distribution of merchandise as holiday bonus to employees, involves the receipt or use of cash. This will be regarded as a non cash activity and neither of the transactions would be reported by Deed Co. in its in its statement of cash flows.Options (a), (b) and (c) are incorrect based on the above explanation. These are non cash activities and will not be shown as inflows or outflows under operating, investing or financing activity sections.
On December 1, year 2, Shine Co. agreed to sell a business segment on March 1, year 3. Throughout year 2, the segment had operating losses that were expected to continue until the segment’s disposition. However, the gain on disposition was expected to exceed the segment’s total operating losses in year 2 and year 3. The amount of estimated net gain from disposal recognized in year 2 equals
A
Zero.
B
The entire estimated net gain.
C
All of the segment’s year 2 operating losses.
D
The segment’s December year 2 operating losses.
A ZERO
Explanation:
In the period in which a component of an entity either has been disposed of or is classified as held for sale, the income statement of a business enterprise for current and prior periods shall report the results of operations of the component…in discontinued operations in the period(s) in which they occur.
Jordan Co. had the following gains during the current period:
Gain on disposal of a business segment $500,000
Foreign currency translation gain 100,000
What amount of gains should be presented on Jordan’s income statement as income from continuing operations?
A $0 B $100,000 C $500,000 D $600,000
A
Explanation:
Gain on disposal of business segments will be reported as discontinued operations and foreign currency translation gains will be reported in other comprehensive income. Therefore, neither of the items will be reflected in income from continuing operations.
Option (B) is incorrect because it will be a part of other comprehensive income.
Option (C) is incorrect because it should be reported as a discontinued operation.
Option (D) is incorrect as per the above explanation.
In hospital accounting, restricted funds are
A
Not available unless the board of directors remove the restrictions.
B
Restricted as to use only for board-designated purposes.
C
Not available for current operating use; however, the income generated by the funds is available for current operating use.
D
Restricted as to use by the donor, grantor, or other source of the resources.
D
Explanation:
In hospital accounting, restricted funds account for financial resources that are externally restricted by donors and grantors for specified operating or research, capital outlay, or endowment purposes. The board of directors of a hospital cannot remove restrictions on the use of financial resources imposed by donors and grantors. While unrestricted resources may be appropriated or designated by the governing board of a hospital for special uses, the board nevertheless has the authority to rescind such actions. Therefore, board-designated assets of a hospital are accounted for in the General Fund.The income generated by a restricted fund may or may not be available for current operating use, depending upon the restrictions imposed upon such income by the donor or grantor.
Gains resulting from the process of translating a foreign entity's financial statements from the functional currency, which has not experienced significant inflation, to U.S. dollars should be included as a (an) A Other comprehensive income item B Deferred credit C Component of income from continuing operations D Extraordinary item
A
Explanation:
If an entity’s functional currency is the foreign currency, and it has not experienced significant inflation, translation adjustments result from the process of translating that entity’s financial statements into the reporting currency. Translation adjustments (gains or losses) are not included in net income, but in other comprehensive income.
Which of the following is true regarding the comparison of managerial to financial accounting?
A
Managerial accounting is generally more precise.
B
Managerial accounting has a past focus and financial accounting has a future focus.
C
The emphasis on managerial accounting is relevance and the emphasis on financial accounting is timeliness.
D
Managerial accounting need not follow generally accepted accounting principles (GAAP) while financial accounting must follow them.
D
Explanation:
Financial and managerial accounting both provide information useful for decision making. Besides differences in primary user, external for financial and internal for managerial, financial accounting must follow generally accepted accounting principles (GAAP), whereas managerial accounting does not.
Option (A) is incorrect because financial accounting is generally more precise than managerial accounting.
Option (B) is incorrect because financial accounting has a past focus rather than managerial accounting.
Option (C) is incorrect because the emphasis in financial accounting is relevance whereas the emphasis in managerial accounting is timeliness.
In preparing its cash flow statement for the current year ended December 31, Reve Co. collected the following data: Gain on sale of equipment $(6,000) Proceeds from sale of equipment 10,000 Purchase of A.S., Inc. bonds (par value $200,000) (180,000) Amortization of bond discount 2,000 Dividends declared (45,000) Dividends paid (38,000) Proceeds from sale of treasury stock (carrying amount $65,000) 75,000In its current year December 31 statement of cash flows, what amount should Reve report as net cash used in investing activities? A $170,000 B $176,000 C $188,000 D $194,000
A
Explanation:
If an exam question does not specify that a debt or equity investment is a cash equivalent or classed as a trading security, then the cash flows from the purchase, sale, or maturity should be classed as cash flows from investing activities.
Purchase of bond investment $180,000
Proceeds from sale of equipment (10,000)
Net cash used in investing activities $170,000
Which of the following assumptions means that money is the common denominator of economic activity and provides an appropriate basis for accounting measurement and analysis?
A Going concern B Periodicity C Monetary unit D Economic entity
C
Explanation:
Monetary unit assumption implies that the economic activity should be measured in money.
Option (A) is incorrect because the going concern concept assumes that the entity will continue to operate in the foreseeable future.
Option (B) is incorrect because periodicity means the economic activity being divided into meaningful time periods.
Option (D) is incorrect because economic entity is defined as an identifiable set of activities.
During a period of inflation in which an asset account remains constant, which of the following occurs?
A
A purchasing power gain, if the item is a monetary asset.
B
A purchasing power gain, if the item is a nonmonetary asset.
C
A purchasing power loss, if the item is a monetary asset.
D
A purchasing power loss, if the item is a nonmonetary asset.
C
Explanation:
Purchasing power losses result from holding monetary assets during a period of inflation because the fixed amount of money will purchase fewer goods and services following a period of inflation. The holding of nonmonetary items during a period of changing prices does not give rise to purchasing power gains or losses.
In determining earnings per share, interest expense, net of applicable income taxes, on convertible debt that is dilutive should be
A
Added back to net income for basic earnings per share, and ignored for diluted earnings per share.
B
Ignored for basic earnings per share, and added back to net income for diluted earnings per share.
C
Deducted from net income for basic earnings per share, and ignored for diluted earnings per share.
D
Deducted from net income for both basic earnings per share and diluted earnings per share.
B
Explanation:
Convertible debt is not included in the calculations of basic EPS. The interest expense is added back to net income in determining dilutive EPS.